You are on page 1of 34

Join Our Telegram Channel https://t.

me/UPSCMaterials For Instant Updates

VISIONIAS
www.visionias.in
ANSWERS & EXPLANATION
GENERAL STUIDES (P) ABHYAAS TEST-1 – 2879 (2019)

Q 1.A
 The Directive Principles of State Policy are contained in the Part IV (Article 36-51) of the
Constitution of India.
 Statement 1 is correct: The framers of the Constitution borrowed this idea from the Irish Constitution of
1937. Dr B R Ambedkar described these principles as ‗novel features‘ of the Indian Constitution.
 Statement 2 is correct: Courts can not declare a law to be invalid on the grounds that it violates a
Directive principle, which means that the Directive Principles are non-justiciable in nature, that is, they
are not legally enforceable by the courts for their violation. Therefore, the Government (Central, state and
local) cannot be compelled to implement them. Nevertheless, the Constitution (Article 37) itself says that
these principles are fundamental in the governance of the country and it shall be the duty of the State to
apply these principles in making laws.
 Statement 3 is not correct: DPSPs require legislation for their implementation. They are not
automatically enforced. For example, the Code of Criminal Procedure 1973 was enacted to give force to
Article 50 (separate the judiciary from the executive in the public services of the state).

Q 2.D
 In formulating the recommendations in respect of the level of minimum support prices and other non-
price measures, the Commission for Agricultural Costs and Prices (CACP) takes into account, apart from
a comprehensive view of the entire structure of the economy of a particular commodity or group of
commodities, the following factors:- Cost of production, Changes in input prices, Input-output price
parity, Trends in market prices, Demand and supply, Inter-crop price parity, Effect on industrial cost
structure, Effect on cost of living, Effect on general price level , International price situation, Parity
between prices paid and prices received by the farmers, Effect on issue prices and implications for
subsidy. So multiple factors are taken into consideration. Hence statement 1 is not correct.
 It is recommended by CACP while approved by Cabinet committee on Economic Affairs. Hence
statement 2 is correct.
 Minimum Support Price (MSP) is a form of market intervention by the Government of India to insure
agricultural producers against any sharp fall in farm prices. The minimum support prices are announced
by the Government of India at the beginning of the sowing season for certain crops on the basis of the
recommendations of the Commission for Agricultural Costs and Prices (CACP). While states may declare
bonus on and above the Minimum Support Prices (MSPs). Hence statement 3 is not correct.

Q 3.C
 The Vernacular Press Act was introduced on 14th March 1878 by the then Governor Lord Lytton. It is
introduced for keeping better control on Indian language newspapers, the act was intended to prevent the
vernacular press from expressing criticism of British policies—notably, the opposition that had grown
with the outset of the Second Anglo-Afghan War (1878–80). The act excluded English-language
publications. Hence statement 1 is not correct.
 The provisions of the Act are given below:
o The District Magistrate was empowered to call upon the printer and publisher of any vernacular
newspaper to enter into a bond with the Government undertaking not to cause disaffection against the
1 www.visionias.in ©Vision IAS
Join Our Telegram Channel https://t.me/UPSCMaterials For Instant Updates

government or antipathy between persons of different religions, caste, race through published
material; the printer and publisher could also be required to deposit security which could be seized if
the offences reoccurred.
o The Magistrate's action was final and no appeal could be made in a court of law. Hence statement 2
is correct.
o A vernacular newspaper could get an exemption from the operation of the Act by submitting proof to
a government censor.
 The Act was finally repealed by Lord Ripon in 1881. Hence statement 3 is not correct.

Q 4.B
 The Union Cabinet recently approved the restructuring of existing National Health Agency as "National
Health Authority" for better implementation of Pradhan Mantri - Jan Arogya Yojana (PM-JAY).
With this approval, the existing society "National Health Agency" has been dissolved and will be replaced
by National Health Authority as an attached office to the Ministry of Health & Family Welfare.
 The National Health Authority (NHA) will provide overall vision and stewardship for design, roll-out,
implementation and management of Pradhan Mantri Jan Arogya Yojana (PM-JAY) in alliance with state
governments. Inter-alia, this will include, formulation of PM-JAY policies, development of operational
guidelines, implementation mechanisms, coordination with state governments, monitoring and oversight
of PM-JAY amongst others.
 The National Health Authority will play a critical role in fostering linkages as well as the convergence of
PM-JAY with health and related programs of the Central and State Governments, including but not
limited to Ayushman Bharat - Comprehensive Primary Health Care, the National Health Mission,
Rashtriya Swasthya Bima Yojana to name a few.

Q 5.D
 The Caspian Sea, located in western Asia on the eastern edges of Europe, is the largest lake on the planet.
The measured surface area is 371,000 sq km (143,244 sq mi), and the maximum depth is at 1025 m
(3,363ft).
 The countries of Azerbaijan, Russia, Kazakhstan, Turkmenistan and Iran border the sea. Oil and
natural gas production platforms are replete along the edges of the sea. In addition, large quantities of
sturgeon live in its waters, and the caviar produced from their eggs is a valuable commodity.
 Fresh water flows into the sea via the Volga River and Ural River in the north, however, the sea remains
somewhat salty in its central and southern parts.

2 www.visionias.in ©Vision IAS


Join Our Telegram Channel https://t.me/UPSCMaterials For Instant Updates

Q 6.D
 Biochemical oxygen demand (BOD) is the amount of dissolved oxygen used by microorganisms in the
biological process of metabolizing organic matter in water. The more organic matter there is (e.g., in
sewage and polluted bodies of water), the greater the BOD; and the greater the BOD, the lower the
amount of dissolved oxygen available for higher animals such as fishes. The BOD is, therefore, a reliable
gauge of the organic pollution of a body of water. One of the main reasons for treating wastewater prior to
its discharge into a water resource is to lower its BOD—i.e., reduce its need of oxygen and thereby lessen
its demand from the streams, lakes, rivers, or estuaries into which it is released. Thus, low BOD would
mean lower organic matter. Due to this, the microbial explosion in the pond will be low. Hence,
statement 1 and 3 is not correct.
 Due to lower BOD, the amount of dissolved oxygen for the higher fishes is high as the oxygen would
not be used by microbes for the process of breaking down of organic matter. Hence statement 2 is not
correct.

Q 7.B
 Raman effect refers to change in the wavelength of light that occurs when a light beam is deflected
by molecules. When a beam of light traverses a dust-free, transparent sample of a chemical compound, a
small fraction of the light emerges in directions other than that of the incident (incoming) beam. Most of
this scattered light is of unchanged wavelength. A small part, however, has wavelengths different from
that of the incident light; its presence is a result of the Raman effect. The phenomenon is named after
Indian physicist Sir Chandrasekhara Venkata Raman, who first published observations of the effect in
1928. Raman spectroscopy, an increasingly popular technique used in many areas including biology
and medicine is based on Raman scattering.
 The scattering of visible light by colloidal particles is called Tyndall Effect.

Q 8.D
 Pair 1 is correctly matched: Houthi rebellion, Sa'dah War, or Sa'dah conflict, was a military rebellion
pitting Zaidi Houthis against the Yemeni military that began in Northern Yemen and has since escalated
into a full-scale civil war.
 Pair 2 is correctly matched: The War in Darfur or Land Cruiser War is a major armed conflict in the
Darfur region of Sudan that began in 2003 when the Sudan Liberation Movement (SLM) and the Justice
and Equality Movement (JEM) rebel groups began fighting the government of Sudan.
 Pair 3 is correctly matched: The War in Donbass is an armed conflict in the Donbass region of Ukraine.
From the beginning of March 2014, protests by pro-Russian and anti-government groups took place in the
Donetsk and Luhansk oblasts of Ukraine, commonly collectively called the "Donbass", in the aftermath of
the 2014 Ukrainian revolution and the Euromaidan movement.

Q 9.A
 The 69th Constitutional Amendment Act of 1991 provided special status to the Union Territory of Delhi,
and redesignated it as the National Capital Territory of Delhi and designated the administrator of Delhi as
the Lieutenant (Lt) Governor. It led to the insertion of two new articles in the form of Article 239AA and
Article 239AB. The following provisions were made under them. It created a legislative assembly and a
council of ministers for Delhi.
 There shall be a Legislative Assembly for the National Capital Territory of Delhi and the seats in such
Assembly shall be filled by members chosen by direct election from territorial constituencies in the
National Capital Territory. Subject to the provisions of the Constitution, the Legislative Assembly shall
have power to make laws for the whole or any part of the National Capital Territory with respect to any of
the matters enumerated in the State List or in the Concurrent List in so far as any such matter is applicable
to Union territories except matters with respect to Entries 1(Public order),2(Police), and 18(Land) of the
State List. But in case, a law made by Legislative Assembly for the National Capital Territory of Delhi is
in opposition to the laws made by Parliament, the laws of Parliament prevail over those made by the
Assembly. Hence, statement 1 is correct.

3 www.visionias.in ©Vision IAS


Join Our Telegram Channel https://t.me/UPSCMaterials For Instant Updates

 The Chief Minister is appointed by the President (not by the Lt. Governor). The other ministers are
appointed by the President on the advice of the Chief Minister. The ministers hold office during the
pleasure of the President. Hence, statement 2 is not correct.
 According to Article 239AA (4), the Council of Ministers headed by the Chief Minister aid and advise
the Lieutenant Governor in the exercise of his functions except in so far as he is required to act in his
discretion. In the case of difference of opinion between the lt. Governor and his ministers, the lt. Governor
is to refer the matter to the President for decision and act accordingly (also mentioned in Article
239AA(4). Hence, statement 3 is not correct.

Q 10.B
 To secure the active cooperation of Indians in the Second World War, the British Government sent to
India a mission headed by a Cabinet minister Stafford Cripps in 1942. The main proposals of the
mission were as follows:
o An Indian Union with a dominion status would be set up; it would be free to decide its relations
with the Commonwealth and free to participate in the United Nations and other international bodies.
o After the end of the war, a constituent assembly would be convened to frame a new constitution.
Members of this assembly would be partly elected by the provincial assemblies through proportional
representation and partly nominated by the princes.
o The British government would accept the new constitution subject to two conditions: (i) any province
not willing to join the Union could have a separate constitution and form a separate Union, and (ii) the
new constitution- making body and the British government would negotiate a treaty to effect the
transfer of power and to safeguard racial and religious minorities.
o In the meantime, defence of India would remain in British hands and the governor-general‘s
powers would remain intact.
 Nehru and Maulana Azad were the official negotiators for the Congress.
 The Cripps Mission proposals failed to satisfy Indian nationalists. The Congress objected to - (i) the offer
of dominion status instead of a provision for complete independence; (ii) representation of the princely
states by nominees and not by elected representatives (iii) right to provinces to secede as this went against
the principle of national unity. Gandhi described the scheme as “a post-dated cheque”. Nehru pointed
out that the ―existing structure and autocratic powers would remain and a few of us will become the
viceroy‘s liveried camp followers and look after canteens and the like‖.

Q 11.B
 Indian classical music has two major traditions - the North Indian classical music tradition is called
Hindustani, while the South Indian expression is called Carnatic. In Haripala's "Sangeeta Sudhakara",
written in the 14th century A.D., the terms Carnatic and Hindustani are found for the first time. The
Indian music of the Northern part of India assimilated some features of the music of the Persian
and Arabic musicians who adorned the courts of the Mughal rulers of Delhi, the music of the South
continued to develop along its own original lines. But the fundamental aspects of both the systems of
the North and South have been the same. Hence statement 1 is correct.
 Kathak is the only form of classical dance wedded to Hindustani or the North Indian music. Both of
them have had a parallel growth, each feeding and sustaining the other.
 The music that accompanies the Kuchipudi dance is the classical school of Carnatic music and is
delightfully syncopatic. The accompanying musicians, besides the vocalist, are a mridangam player to
provide percussion music, a violin or veena player or both for providing instrumental melodic music, and
a cymbal player who usually conducts the orchestra and recites the sollukattus (mnemonic rhythm
syllables).
 The repertoire of Mohiniyattam includes music in the Carnatic style, singing and acting a play
through the dance, where the recitation may be either by a separate vocalist or the dancer herself. The
jatiswaram of Bharatanatyam is a short pure dance piece performed to the accompaniment of
musical notes of any raga of Carnatic music. Kathakali music uses Carnatic ragas - the raga and tala
conforming to the bhava, rasa and dance patterns (nritta and natya). Hence statement 3 is correct.
 In contrast to northern styles, Carnatac music is more thoroughly oriented to the voice. Even when
instruments are used alone, they are played somewhat in imitation of singing, generally within a vocal

4 www.visionias.in ©Vision IAS


Join Our Telegram Channel https://t.me/UPSCMaterials For Instant Updates

range, and with embellishments that are characteristic of vocal music. Fewer instruments are used in
Carnatic than in northern Indian music, and there are no exclusively instrumental forms. Although
vocal music plays an important role, instrumental music is more important in Hindustani music than it is
in Carnatac; there are some purely instrumental forms, such as the theme with variations known as
gat. Hence statement 2 is not correct.

Q 12.D
 Indravati National Park is a national park located in Bijapur district of Chhattisgarh state of India. It
is home to one of the last populations of rare wild buffalo and hill mynas. It is one among the two project
tiger sites in Chhattisgarh along with Udanti-sitanadi, Indravati National Park is located in Bijapur district
of Chhattisgarh. The park derives its name from the Indravati River, which flows from east to west
and forms the northern boundary of the reserve with the Indian state of Maharashtra. With a total area of
approximately 2799.08 km2, Indravati attained the status of a national park in 1981 and a tiger reserve in
1983 under the famous Project Tiger of India, to become one of the most famous tiger reserves of
India. The vegetation is mainly deciduous type consisting of Sal, Teak and bamboo trees. The
grasslands of Indravati National Park support wild animals like buffaloes, Chital, Barking Deer, Nilgai,
and Gaurs. Some other animals found in the region are Sambar, Sloth Bear, Dhole (Wild Dog), Striped
Hyena, Muntjac, Wild Boar, Flying Squirrel, Porcupine, Pangolins, etc. The Park also shelters a lot of
reptiles like crocodiles, Monitor Lizard, Indian Chameleon, Common Krait and Indian Rock
python. Hence option (d) is the correct answer.
 Panna National Park is a national park located in Panna and Chhatarpur districts of Madhya Pradesh in
India. The region, famous for its diamond industry, is also home to some of the best wildlife species in
India and is one of the most famous Tiger Reserves in the country.
 Pench National Park is one of the premier tiger reserves of India and the first one to straddle across two
states - Madhya Pradesh and Maharashtra. It is a residence of Bengal Tigers accompanied by Chital,
Jungle Cat, Wolf, Indian Leopard, Gaur, Four-horned Antelope, Sloth Bear and much other wildlife. Since
the national park is also the abode of the national animal- Tigers, this park has been included under
Project Tiger and hence is a tiger reserve since 1992.
 Madhav National Park is located in Shivpuri district of Madhya Pradesh state in Central India. It is in
the northern region of Madhya Pradesh. This park boasts some of the most exquisite species like the little
Chinkara, Nilgai, Sambar, Chausingha and deer species such as Chital, Sambhar and barking deer.

Q 13.A
 Bangladesh and India share a 4156 Km. long international border. State-wise length of the border is as
follows –
o West Bengal- 2217 Km, Tripura-856 Km, Meghalaya – 443 Km, Assam -262 Km and Mizoram –
180Km. Hence statement 1 is correct.
 Sonbhadra is a district of Uttar Pradesh. It is the only district in India which shares its boundary with four
different states – Chhattisgarh, Madhya Pradesh, Jharkhand, Bihar. Hence statement 2 is correct.
 The River Krishna forms the border between the states of Telangana and Andhra Pradesh from Srisailam
to Pulichintala for about 290 km flowing through Nagarjuna Sagar Dam. Hence statement 3 is not
correct.

Q 14.A
 Tariff is a tax on imports and exports. Export duties and import duties are classified as tariff barriers.
 A non-tariff barrier is a form of non-tax restrictions trade where barriers to trade are set up and take a
form other than a tariff. It includes quotas, embargoes, sanctions, levies and other restrictions and is
frequently used by large and developed economies. Non-tariff barriers include: Import bans; General or
product-specific quotas; Complex/discriminatory Rules of Origin; Unjustified Sanitary and Phyto-
sanitary conditions, Unreasonable/unjustified packaging, labelling, product standards; Complex
regulatory environment; Additional trade documents like Certificate of Origin, Certificate of Authenticity
etc.; Occupational safety and health regulation; Import licenses; State subsidies; Export subsidies;
Fixation of a minimum import price among others.
 Hence, the correct answer is option (a).

5 www.visionias.in ©Vision IAS


Join Our Telegram Channel https://t.me/UPSCMaterials For Instant Updates

Q 15.C
 Recently World Bank endorsed a new Country Partnership Framework (CPF) for India. The CPF
aims to support India‟s transition to a higher middle-income country by addressing some of its key
development priorities - resource efficient and inclusive growth, job creation and building its
human capital.
 The World Bank Group‘s Country Partnership Framework (CPF) aims to make the country-driven model
more systematic, evidence-based, selective and focused on the Bank‘s twin goals of ending extreme
poverty and increasing shared prosperity in a sustainable manner. The CPF replaces the Country
Assistance Strategy (CAS). Used in conjunction with a Systematic Country Diagnostic (SCD), the CPF
guides the World Bank Group‘s (WBG) support to a member country.

Q 16.C
 Nipah is a virus which commonly affects animals like bats, pigs, dogs, horses, etc. The virus can spread
from animals to humans (zoonotic) and can sometimes cause serious illness among humans.
 Fruit bats of the genus Pteropus are the natural reservoirs of NiV. Possible routes of transmission of
NiV include consumption of fruit contaminated by the saliva of infected bats, from direct contact with
infected bats or their feces/urine, or human-to-human transmission through unprotected close contact with
an infected patient in the community or hospital.
 The illness commonly presents as brain fever. In some cases during early part of illness, it may also
present with fever with persistent cough and difficulty in breathing.
 Currently, there are no specific treatments available for Nipah virus disease and care is supportive.
Intensive supportive care is recommended to treat severe respiratory and neurologic complications.
 As per the latest report of the Central High-Level Team, Nipah Virus Disease is not a major outbreak
and only local occurrence limited to two districts in Kerala i.e. Kozhikode & Malappuram.
 Hence, only statements 1 and 3 are correct.

Q 17.C
 Annual Status of Education Report (ASER), India‟s largest NGO-run annual survey, has been
conducted by Pratham since 2005 to evaluate the relevance and impact of its programs. Findings are
disseminated at national, state, district and village levels, and influence education policies at both state
and central levels.
 To monitor improvement in children‘s learning levels and to periodically assess the health of the
government education system as a whole, the National Council of Educational Research and Training
(NCERT) has been periodically conducting National Achievement Surveys (NAS) since 2001, for Class
III, V and VIII. The NAS report gives a national and state-level picture, rather than scores for
individual students, schools or districts.
 The differences between ASER survey and National Achievement Survey (NAS) are as follows:
o ASER Survey is a household survey conducted since 2005. It consists of one-on-one oral
assessments. It is aimed at the representative sample of all children (Whether in school or out of
school). It focuses on foundational skills like reading and maths. It is limited to rural areas of the
country. It is a citizen-led survey.
o National Achievement Survey is a school based survey. It consists of pen and paper tests. It takes
into account the children enrolled in government schools and government-aided schools. It looks at a
wider variety of skills not limited to just maths and reading. It is conducted throughout the country in
both rural and urban areas. It is conducted by the NCERT under Ministry of Human Resource
and Development.

Q 18.D
 Quinine: Quinine is a medication used to treat malaria. Quinine comes from the bark of the cinchona
tree. This tree is native to central and South America, as well as some islands in the Caribbean and
western parts of Africa.
 Morphine: Morphine is an opiate, derived from the poppy plant. It is classified as a narcotic and is
commonly prescribed to manage pain. Morphine is often used before or after surgery to alleviate severe
pain. Morphine and other opioids act by attaching to specific proteins called opioid receptors, which are

6 www.visionias.in ©Vision IAS


Join Our Telegram Channel https://t.me/UPSCMaterials For Instant Updates

found in the brain, spinal cord, and gastrointestinal tract. When these compounds attach to certain opioid
receptors in the brain and spinal cord, they can effectively change the way a person experiences pain.
 Penicillin: Penicillin was the first antibiotic to be discovered, and it was a chance discovery. Alexander
Fleming while working on Staphylococci bacteria, once observed a mould (Fungi colony) growing in one
of his unwashed culture plates around which Staphylococci could not grow. He found out that it was due
to a chemical produced by the mould (Fungus) and he named it Penicillin after the mould
Penicillium notatum. However, its full potential as an effective antibiotic was established much later by
Ernest Chain and Howard Florey. This antibiotic was extensively used to treat American soldiers
wounded in World War II.

Q 19.A
 Ibn Battuta‟s book of travels, called Rihla, written in Arabic, provides extremely rich and interesting
details about the social and cultural life in the subcontinent in the fourteenth century.
 Statement 1 is not correct: Ibn Battuta visited Vijayanagara kingdom in 14th century whereas
Krishnadevaraya of the Vijayanagara Empire reigned from 1509–1529.
 Statement 2 is correct: Ibn Battuta was amazed by the efficiency of the postal system which allowed
merchants to not only send information and remit credit across long distances but also to dispatch goods
required at short notice. This is how Ibn Battuta describes the postal system: In India the postal system is
of two kinds. The horse-post, called uluq, is run by royal horses stationed at a distance of every four
miles. The foot-post has three stations per mile; it is called dawa, that is one-third of a mile.
 Statement 3 is not correct: Ibn Battuta found cities in the subcontinent full of exciting opportunities
for those who had the necessary drive, resources and skills. They were densely populated and
prosperous, except for the occasional disruptions caused by wars and invasions.

Q 20.B
 Tropical cyclone is an intense circular storm that originates over warm tropical oceans and is
characterized by low atmospheric pressure, high winds, and heavy rain.
 Cyclones in the northern hemisphere rotate in the anticlockwise direction and those in the southern
hemisphere rotate in the clockwise direction.
 So, the ones that typically strike the Indian neighbourhood in the northern hemisphere rotate
anticlockwise. Their normal behaviour is to derive strength from the moisture in waters such as the Bay of
Bengal, move west, incline in a northerly direction and peter out into the sea or land, depending on their
origin.
 However, a re-curving cyclone is deflected to the right(eastwards) in the northern hemisphere.
o This is due to air currents in the local atmosphere that push cold air from the poles towards the
equator and interfere with cyclone formation. In the southern hemisphere, the cyclones spin clockwise
and therefore also re-curve in the opposite direction.
 During the monsoon months, cyclones in the Western Pacific move westwards towards India and aid the
associated rain-bearing systems over the country. However, in the years of a re-curve, they do not give as
much of a push to the rain.
 Long-term data suggest that while there has been an increase in the number of tropical cyclones in India‘s
neighbourhood there is no clear trend in re-curving ones. In general, cyclone activity in India peaks
around November, by which time, the summer monsoon has already passed.
 Cyclone Ockhi, that formed in November 2017, was among the rare re-curving cyclones with a
presence over the Arabian Sea. According to the IMD( India Meteorological Department), a severe
cyclonic storm in 1912 took an almost identical route from Sri Lanka to the coast of Maharashtra.
 A challenge with re-curving cyclones is that it is hard for weather models to pick them early on — as was
the case with Ockhi — and so they pose unique challenges in terms of hazard preparedness and disaster
management.

Q 21.A
 Article 32 deals with right to move the supreme court for the enforcement of fundamental rights. Locus
Standi is mandatory for most of the provisions under Article 32 while in few cases, Locus standi (which

7 www.visionias.in ©Vision IAS


Join Our Telegram Channel https://t.me/UPSCMaterials For Instant Updates

means the ability of a party to demonstrate to the court sufficient connection to and harm from the
law or action challenged to support that party's participation in the case) is not necessary.
 Statement 1 is not correct: Before 1980s, only the affected parties had the locus standi (standing
required in law) to file a case and continue the litigation and the non affected persons had no locus standi
to do so. But after 1980s, the traditional rule of locus standi was liberalized to give way to Public Interest
Litigation (PIL). Through PIL, the Court increased the concept of 'person aggrieved' to include any
public-spirited individual or association, provided they act for proper cause of justice, and are not actuated
by political motives or other forces. Now, a third party appeal can be made to seek justice on
violations of rights and it should be noted that only concept of aggrieved party has been enlarged,
not the basic part.
 Statement 2 is correct: A PIL cannot be filed for matters dealing with petitions for early hearing of
cases in High Courts and Subordinate Courts. Other cases that cannot be entertained through PIL are:
o Landlord-tenant matters.
o Service matter and those pertaining to pension and gratuity.
o Admission to medical and other educational institution.
 Statement 3 is not correct: Judges themselves have in some cases initiated suo moto actionbased on
newspaper articles or letters received. For instance, in May 2017 taking cognizance of a letter issued by
Law Ministry to Secretary General of Supreme Court, a three Judge Bench of Supreme Court of
India issued notice in a suo motu PIL to state governments seeking their response on Central Selection
Mechanism for Subordinate judiciary to make selections to the subordinate judiciary across the Country.

Q 22.A
 French President Emmanuel Macron announced the “Paris Call for Trust and Security in
Cyberspace,” a new initiative designed to establish international norms for the internet, including
good digital hygiene and the coordinated disclosure of technical vulnerabilities, during the annual
UNESCO Internet Governance Forum in Paris.
 More than 190 signatures were obtained on the Paris Call, including 130 from private sector and more
than 50 member nations. However, India and US didn‘t sign the agreement. Russia, China, Iran, and Israel
didn't sign, either. Some of the abstainers, like China and Iran, have active cyberwar initiatives. The Paris
Call is not legally binding on its signatories which raises questions on its efficacy. Putting in place a
legally binding compliance mechanisms is one of the issues that have to be sorted out to create a more
secure cyberspace. It doesn‘t require governments or corporations legally adhere to any specific
principles. It‘s mostly a symbol of the need for diplomacy and cooperation in cyberspace, where it‘s hard
to enforce any single country‘s laws.
 Major American technology corporations including Microsoft, Facebook, Google, IBM, and HP all
endorsed the agreement. Influential non-governmental groups like World Leadership Alliance, Chatham
House, Carnegie Endowment for International Peace, the World Wide Web Foundation and the Internet
Society have supported too.

Q 23.A
 Voluntary Unemployment- Voluntary unemployment is defined as a situation where the unemployed
choose not to accept a job at the going wage rate. The individual in the question experiences this after
having quit the R&D company. Thus he remains voluntarily unemployed. Hence, option 1 is correct.
 Underemployment- It is a situation where a highly skilled individual is working in a low-paying or low-
skilled job. The individual in the context, who is a PhD, works as an agricultural labourer at his relative's
farm which is inferior to his qualifications and skills. Hence, option 2 is correct. At the same time, he
cannot be said to have undergone disguised unemployment (A situation where a part of the labor force is
either left without work or is working in a redundant manner where worker productivity is essentially
zero) as he contributes in tripling his relative's income. Hence, option 4 is not correct.
 Frictional unemployment- Frictional unemployment is a type of unemployment that arises when workers
are searching for new jobs or are transitioning from one job to another. It is a part of natural
unemployment and hence is present even when the economy is considered at full employment.

8 www.visionias.in ©Vision IAS


Join Our Telegram Channel https://t.me/UPSCMaterials For Instant Updates

The individual in the context undergoes this twice, first during the intervening period between his job
at his relative's farm and with the R&D company and second, after quitting the R&D company (where he
also experiences voluntary unemployment). Hence, option 3 is correct.

Q 24.D
 Statement 1 is not correct. 73rd Constitutional Amendment Act of 1992 gives practical shape to idea of
Village Panchayats and made it constitutionally mandatory on the part of states to establish institutions of
local self governance. However, it only added eleventh schedule to the Constitution. Twelfth schedule to
the Constitution was added through 74rd Constitutional Amendment Act of 1992.
 Statement 2 is not correct. The act applies to the whole of India except states of Jammu and Kashmir,
Nagaland, Meghlaya and Mizoram and certain other areas. These areas include:
o Hill areas in the State of Manipur for which District Councils
o The district level provisions shall not apply to the hill areas of the District of Darjeeling in the State of
West Bengal which affect the Darjeeling Gorkha Hill Council.
o The reservation provisions are not applicable to Arunachal Pradesh.

Q 25.D
 The Perform Achieve Trade (PAT) is an innovative, market-based trading scheme announced by the
Indian Government in 2008 under its National Mission on Enhanced Energy Efficiency (NMEEE) in
National Action Plan on Climate Change (NAPCC).
 Statement 3 is correct: The scheme is being designed and implemented by the Bureau of Energy
Efficiency (BEE), under the Ministry of Power of India.
 The 2010 amendment to the Energy Conservation Act (ECA) provides a legal mandate to PAT.
 Statement 1 is correct: It aims to improve energy efficiency in industries by trading in energy efficiency
certificates in energy-intensive sectors.
o The scheme provides the option to trade any additional certified energy savings with other designated
consumers to comply with the Specific Energy Consumption reduction targets.
o Facilities making greater reductions than their targets receive ―EsCerts‖ or ―energy saving
certificates‖ which can be traded with facilities that are having trouble meeting their targets, or
banked for future use.
o The Energy Savings Certificates (ESCerts) so issued will be tradable on special trading platforms to
be created in the two power exchanges -- Indian Energy Exchange and Power Exchange India.
 Statement 2 is correct: The scheme imposes mandatory specific energy consumption targets on the
covered facilities with less energy efficient facilities having a greater reduction target than the more
energy efficient ones. Participation in the scheme is mandatory for certain designated consumers under the
ECA. The PAT scheme establishes plant-specific targets rather than a sectoral target.
o Mandatory, specific targets for energy consumption for larger, energy-intensive facilities are set by
BEE.
o The PAT Scheme is being implemented in three phases
 The first phase ran from 2012-2015 covering 478 facilities from eight energy-intensive sectors,
namely aluminum, cement, chor-alkali, fertilizer, iron and steel, pulp and paper, textiles and
thermal power plants.
 This accounts for roughly 60% of India‘s total primary energy consumption. It targets energy
consumption reductions of 6.6 million tons of oil equivalent in the 478 covered facilities.

Q 26.D
 Statement 1 is correct: The criteria for declaring a language as classical mandates high antiquity of its
early texts/recorded history over a period of 1,500-2,000 years, a body of ancient literature/texts which is
considered a valuable heritage by generations of speakers and a literary tradition that is original and not
borrowed from another speech community. Also since the classical language and literature is distinct from
the modern, there can also be a discontinuity between the classical language and its later forms or its
offshoots.
 Statement 2 is correct: Once a language is declared classical, it gets financial assistance for setting up a
centre of excellence for the study of that language and also opens up an avenue for two major awards for

9 www.visionias.in ©Vision IAS


Join Our Telegram Channel https://t.me/UPSCMaterials For Instant Updates

scholars of eminence. Besides, the University Grants Commission can be requested to create – to begin
with at least in Central Universities – a certain number of professional chairs for classical languages for
scholars of eminence in the language.
 Odia became the sixth language of the country to get ―classical language‘‘ status after the Union Cabinet
conceded a long-pending demand for putting it in the same league as Sanskrit, Tamil, Telugu, Kannada
and Malayalam. Odia is billed as the first language from the Indo-Aryan linguistic group.
 Statement 3 is correct: All the six languages which have been given the status of classical language
come under the Eighth Schedule of the Indian Constitution. The eighth schedule consists of 1) Assamese,
(2) Bengali, (3) Gujarati, (4) Hindi, (5) Kannada, (6) Kashmiri, (7) Konkani, (8) Malayalam, (9)
Manipuri, (10) Marathi, (11) Nepali, (12) Odia, (13) Punjabi, (14) Sanskrit, (15) Sindhi, (16) Tamil, (17)
Telugu, (18) Urdu (19) Bodo, (20) Santhali, (21) Maithili and (22) Dogri.

Q 27.B
 During the period between 1940-1946, due to the absence of popular ministries, domination of the Indian
scene by political problems and the preoccupation of the government with war efforts, the pace of
educational progress slowed down.
 After the end of Second World War, the Central Advisory Board of Education (CABE) in India published
a comprehensive report on the "Post-War Educational Development in India" in the country. This was
the first systematic and national level attempt to review the problems of education as a whole. It is
also known as Sargent Plan after John Sargent, the then Educational Advisor to the Government of
India. Surprisingly, the report is not known after its chairman (Sir Jogindra Singh), but its one of
the members of the Committee – John Sargent (the educational advisor to the Government of
India).
 It proposed the establishment of nursery schools on a voluntary basis for children under six, while from
six to fourteen years of age education should be free and compulsory for both boys and girls in a phased
program spread over 40 years (1944-1984).

Q 28.C
 The Union Budget, 2018-19 proposed to treat school education holistically without segmentation from
pre-nursery to Class 12. Samagra Shiksha - an overarching programme for the school education
sector extending from pre-school to class 12 has been, therefore, prepared with the broader goal of
improving school effectiveness measured in terms of equal opportunities for schooling and equitable
learning outcomes. It subsumes the three Schemes of Sarva Shiksha Abhiyan (SSA), Rashtriya
Madhyamik Shiksha Abhiyan (RMSA) and Teacher Education (TE).
 The scheme envisages the ‗school‘ as a continuum from pre-school, primary, upper primary, secondary to
Senior Secondary levels. The vision of the Scheme is to ensure inclusive and equitable quality education
from pre-school to senior secondary stage in accordance with the Sustainable Development Goal (SDG)
for Education.
 The major objectives of the Scheme are:
o Provision of quality education and enhancing learning outcomes of students;
o Bridging Social and Gender Gaps in School Education;
o Ensuring equity and inclusion at all levels of school education;
o Ensuring minimum standards in schooling provisions;
o Promoting Vocationalisation of education;
o Support States in the implementation of Right of Children to Free and Compulsory Education
(RTE) Act, 2009; and
o Strengthening and up-gradation of SCERTs/State Institutes of Education and DIET as a nodal
agencies for teacher training.
 The main outcomes of the Scheme are envisaged as Universal Access, Equity and Quality, promoting
Vocationalisation of Education and strengthening of Teacher Education Institutions (TEIs).
 The Scheme will be implemented as a Centrally Sponsored Scheme by the Department of School
Education and Literacy through a single State Implementation Society (SIS) at the State/UT level. At the
National level, there would be a Governing Council headed by Minister of Human Resource Development
and a Project Approval Board (PAB) headed by Secretary, Department of School Education and Literacy.

10 www.visionias.in ©Vision IAS


Join Our Telegram Channel https://t.me/UPSCMaterials For Instant Updates

 The fund sharing pattern for the scheme between Centre and States is at present in the ratio of 90:10 for
the 8 North-Eastern States viz. Arunachal Pradesh, Assam, Manipur, Meghalaya, Mizoram, Nagaland,
Sikkim and Tripura and 3 Himalayan States viz. Jammu & Kashmir, Himachal Pradesh and Uttarakhand
and 60:40 for all other States and Union Territories with Legislature. It is 100% centrally sponsored for
Union Territories without Legislature. This is in accordance with the recommendations of the Sub-Group
of Chief Ministers on Rationalization of Centrally Sponsored Schemes received in October, 2015.

Q 29.A
 The Shahpur-Kandi project, located in Gurdaspur district of Punjab, involves construction of a
55.5-m-high dam on Ravi river. It aims at providing irrigation facility to 5,000 hectares of land in
Punjab and 32,173 hectares in Jammu and Kashmir, besides generating 206 MW of electricity.
 The Ratle Hydroelectric Plant is a run-of-the-river hydroelectric power station currently under
construction on the Chenab River in Kishtwar district of the Indian state of Jammu and Kashmir.
 The Nimoo Bazgo Power Project is a run-of-the-river power project on the Indus River situated
near Leh in Jammu and Kashmir.
 The Kishanganga Hydroelectric Plant is a run-of-the-river hydroelectric scheme that is designed to
divert water from the Kishanganga River to a power plant in the Jhelum River basin. It is located in
Jammu and Kashmir, India and will have an installed capacity of 330 MW.
 Some other important Projects on Indus river system in news are:
o Baglihar Hydroelectric Power Project: It is a run-of-the-river power project on the Chenab River in
the southern Doda district of Jammu and Kashmir. This project was conceived in 1992, approved in
1996 and construction began in 1999. Ujh is a tributary of Ravi river in Kathua district of Jammu and
Kashmir. It was revived in 2017, with the Central Water Commission having prepared and submitted
a fresh detailed project report for a multipurpose hydropower project that is likely to generate 186
MW of electricity and provide irrigation to 30,000 hectares of land. The estimated cost is about Rs
5,000 crore. Work has not yet started at the project site.
o Salal Hydroelectric Power Station, is a run-of-the-river power project on the Chenab River in the
Reasi district of Jammu and Kashmir.
o Dul Hasti is a 390 MW hydroelectric power plant on Chenab river in Kishtwar district of
Jammu and Kashmir.
o The Pakal Dul Dam is a proposed concrete-face rock-fill dam on the Marusadar River, a
tributary of the Chenab River, in Kishtwar district of Jammu and Kashmir.

Q 30.C
 The International Seabed Authority (ISA) is comprised of 167 Member States, and the European
Union. All Parties to the 1982 United Nations Convention on the Law of the Sea (UNCLOS) are
members of ISA. ISA is mandated as the organization through which Parties to UNCLOS shall organize
and control all mineral-related activities in the international seabed area, known also as “the Area.”
 The international seabed area - the part which is under ISA jurisdiction - is the seabed and ocean floor and
the subsoil thereof, beyond the limits of national jurisdiction. The international seabed area represents
around 54 percent of the total area of the world‘s oceans.
 Two principal organs establish the policies and govern the work of ISA, the Assembly, where all 168
members are represented, and a 37-member Council elected by the Assembly. The Council has two
advisory bodies, the Legal and Technical Commission (30 members) which advises the Council on all
matters relating to the exploration and exploitation of non-living marine resources, such as polymetallic
nodules, polymetallic sulphides and cobalt-rich ferromanganese crusts, and the Finance Committee (15
members) that deals with budgetary and related matters.
 Part of ISA‘s mandate and policies is to promote the orderly, safe and responsible management and
development of the resources of the deep seabed area for the benefit of mankind as a whole. In doing so,
ISA has the duty to adopt appropriate rules, regulations and procedures to ensure the effective protection
of the marine environment from harmful effects that may arise from mineral exploration and exploitation
in the Area. Another critical aspect of ISA‘s mandate is to promote and encourage the conduct of marine
scientific research in the seabed area as well as coordinate and disseminate the results of such research
and analysis.

11 www.visionias.in ©Vision IAS


Join Our Telegram Channel https://t.me/UPSCMaterials For Instant Updates

Q 31.C
 The 97th Constitutional Amendment Act of 2011 gave a constitutional status and protection to co-
operative societies. In this context, it made the following changes in the Constitution:
o It made the right to form co-operative societies a Fundamental Right (Article 19). Hence, option 1
is correct.
o It included a new Directive Principle of State Policy on promotion of cooperative societies (Article
43-B). Hence, option 2 is correct.
o It added a new Part IX-B in the Constitution which is entitled ―The Cooperative Societies‖ (Articles
243-ZH to 243-ZT).
o Also, the ―co-operative societies‖ is a subject enumerated in Entry 32 of the state list of the Seventh
Schedule of the Constitution and the state legislatures have accordingly enacted legislations on co-
operative societies. But there is no mention of Co-operative Societies in Fundamental Duties. Hence,
option 3 is correct.

Q 32.C
 Primary pollutants from cars and trucks are Particulate Matter, SOX, NOX, Carbon Monoxide, Sulphur
Dioxide and other hazardous air pollutants (toxics) like Benzene, Acetaldehyde and 1,3-butadiene.
 In SCR technology, a nitrogen-based reagent such as AUS 32, which contains ammonia or urea, is mixed
with the outgoing waste gas mixture.
 It converts NOX molecule into molecular Nitrogen and water vapour.

Q 33.B
 Statement 1 is not correct: The foundation of the Buddhist vihara at Sanchi that includes the great
Sanchi Stupa was laid by one of the greatest Indian Emperors, Ashoka of the Maurya Dynasty who
reigned over almost the entire Indian subcontinent from c. 268 to 232 BCE. He commissioned
construction of the Stupa here after redistributing the mortal remains of Lord Buddha so as to build
several Stupas in different locations across India to spread Buddhism.
 Statement 2 is correct: The reliefs of Sanchi display (among other representations) the following quite
prominently:
o The four great events of the Buddha's life, i.e. birth, attainment of knowledge, dharmachakra -
pravartana and Mahaparinirvana.

12 www.visionias.in ©Vision IAS


Join Our Telegram Channel https://t.me/UPSCMaterials For Instant Updates

o Representations of birds and animals like lion, elephant, camel, ox, etc. are abundant. Some of
the animals are shown with riders in heavy coats and boots.
o Lotus and wishing-vines have been prominently and beautifully carved out as ornamentation, and
Unique representation of forest animals in a manner which looks as if the whole animal world turned
out to worship the Buddha.
 Statement 3 is correct: Originally the stupa was made of bricks and surrounded by a wooden
railing. The existing stupa at Sanchi encloses the original stupa and has been enlarged and enclosed
within the stone railing when stone was adopted in the place of wood. To the stupa which consisted of a
domical structure, a base, sometimes circular, sometimes square, was added in the 1st century B.C., a
circumambulatory path as well as the stone railing with four elegantly carved gateways in the four
cardinal directions.

Q 34.A
 The Ring of Fire is a string of volcanoes and sites of seismic activity, or earthquakes, around the edges of
the Pacific Ocean. Roughly 90% of all earthquakes occur along the Ring of Fire, and the ring is dotted
with 75% of all active volcanoes on Earth.Hence statements 1 is correct.
 The Ring of Fire is the result of plate tectonics(convergent plate boundary). Tectonic plates are huge slabs
of the Earth‘s crust, which fit together like pieces of a puzzle. The plates are not fixed but are constantly
moving atop a layer of solid and molten rock called the mantle. Sometimes these plates collide, move
apart, or slide next to each other. Most tectonic activity in the Ring of Fire occurs in these geologically
active zones. A convergent plate boundary is formed by tectonic plates crashing into each other.
Convergent boundaries are often subduction zones, where the heavier plate slips under the lighter plate,
creating a deep trench. This subduction changes the dense mantle material into buoyant magma, which
rises through the crust to the Earth‘s surface. Over millions of years, the rising magma creates a series of
active volcanoes known as a volcanic arc. Hence statement 2 is correct.
 Ring of Fire lies in both northern and southern hemisphere. Hence statement 3 is not correct.

Q 35.C
 Statement 1 is not correct and statement 2 is correct: The Treaty on the Non-Proliferation of Nuclear
Weapons (NPT) is an international treaty whose objective is to prevent the spread of nuclear weapons and
weapons technology, to promote cooperation in the peaceful uses of nuclear energy and to further the goal
of achieving nuclear disarmament and general and complete disarmament. The Treaty represents the only
binding commitment in a multilateral treaty to the goal of disarmament by the nuclear-weapon States.

13 www.visionias.in ©Vision IAS


Join Our Telegram Channel https://t.me/UPSCMaterials For Instant Updates

Opened for signature in 1968, the Treaty entered into force in 1970. On 11 May 1995, the Treaty was
extended indefinitely. A total of 191 States have joined the Treaty, including the five nuclear-weapon
States.
 Statement 3 is correct: The Treaty establishes a safeguards system under the responsibility of the
International Atomic Energy Agency (IAEA). Safeguards are used to verify compliance with the Treaty
through inspections conducted by the IAEA. The Treaty promotes cooperation in the field of peaceful
nuclear technology and equal access to this technology for all States parties, while safeguards prevent the
diversion of fissile material for weapons use.

Q 36.B
 Election Commission of India is a permanent Constitutional Body. The Constitution of India has vested in
the Election Commission of India the superintendence, direction and control of the entire process for
conduct of elections to Parliament and Legislature of every State and to the offices of President and Vice-
President of India.
 Statement 1 is not correct. Election Commission of India(ECI) has only the power of registration of
political parties, not the deregistration or de-recognition powers, for the pupose of election. ECI grants
them the status of national or state parties on the basis of their poll performances. However, The Election
Commission of India (ECI) in an affidavit to the Supreme Court has said it must be empowered to
deregister a political party if it violates provisions of the Constitution.
 Statement 2 is correct. Under the Constitution, the Commission also has advisory jurisdiction in the
matter of post election disqualification of sitting members of Parliament and State Legislatures. Further,
the cases of persons found guilty of corrupt practices at elections which come before the Supreme Court
and High Courts are also referred to the Commission for its opinion on the question as to whether such
person shall be disqualified and, if so, for what period. The opinion of the Commission in all such matters
is binding on the President or, as the case may be, the Governor to whom such opinion is tendered.
 Statement 3 is correct.The Commission has the power to disqualify a candidate who has failed to lodge
an account of his election expenses within the time and in the manner prescribed by law. The Commission
has also the power for removing or reducing the period of such disqualification as also other
disqualification under the law.

Q 37.B
 Karakoram Range: Karakoram Range, with a length of 500 km holds many highest peaks of Earth. K2,
the second highest peak of the world, at 8,611m lies in the Karakoram Range. Hindu-Kush, an extension
of the Karakoram Range runs in Afghanistan.
 Ladakh Range: It is located south of the Karakoram Range.
 The Pir Panjal range: is located in the southern direction of Himalayas starting from Himachal Pradesh
in India and running north-west towards Jammu & Kashmir. It is the longest and most important range of
middle Himalayas. It extends from the Jhelum river to the upper Beas river for over 300 km. It rises to
5,000 metres and contains mostly volcanic rocks.
 Kumaon Hills: Kumaon is a maze of mountains, part of the Himachal range (Middle Himalayas), some
of which are among the loftiest known. They run along the state of Uttarakhand.

14 www.visionias.in ©Vision IAS


Join Our Telegram Channel https://t.me/UPSCMaterials For Instant Updates

Q 38.C
 Land use, land-use change and forestry (LULUCF) is a category within the United Nations Framework
Convention on Climate Change (UNFCCC) accounting framework for greenhouse gas (GHG) emissions.
 It includes the carbon pools of living biomass (above and below ground), dead organic matter (dead wood
and litter) and organic soil carbon.
 Emissions from land-use change (such as permanent pasture to arable) and deforestation are also reported
under this category.
 The land-use sector (managed cropland, forestry etc.) can be responsible for GHG emissions (for example
from deforestation or draining of wetlands) as well as removals of GHG from the atmosphere (notably
CO2 which is absorbed by plant growth and agricultural soil management).
 Harvested wood products can be a store of carbon (for example when used as building materials), but can
also cause GHG emissions (for example when burned as biomass for energy).
 World's forests and other wooded lands store more than 485 gigatonnes (1 Gt=1 billion tonnes) of carbon,
260 Gt in the biomass (53 per cent), 37 Gt in dead wood and litter (8 percent) and 189 Gt in soil (39 per
cent)
 LULUCF sector is a net carbon sink.

Q 39.B
 Organic products are grown under a system of agriculture without the use of chemical fertilizers and
pesticides with an environmentally and socially responsible approach. This is a method of farming that
works at grass root level preserving the reproductive and regenerative capacity of the soil, good plant
nutrition, and sound soil management, produces nutritious food rich in vitality which has resistance to
diseases.
 India is bestowed with lot of potential to produce all varieties of organic products due to its various agro
climatic regions. In several parts of the country, the inherited tradition of organic farming is an added
advantage. This holds promise for the organic producers to tap the market which is growing steadily(both
the domestic and export markets).
 India‟s rank in terms of world‟s organic agricultural land was 9th and in terms of total number of
producers was 1st as per 'The World of Organic Agriculture Statistics and Emerging Trends 2019'
report.
 The Government of India has implemented the National Programme for Organic Production (NPOP). The
national programme involves the accreditation programme for certification Bodies, standards for organic
production, promotion of organic farming etc.
 India produced around 1.70 MT (2017-18) of certified organic products which includes all varieties of
food products namely oil seeds, sugar cane, cereals & millets, cotton, pulses, medicinal plants, tea, fruits,
spices, dry fruits, vegetables, coffee etc. The production is not limited to the edible sector but also
includes organic cotton fiber, functional food products etc.
 Among different states Madhya Pradesh is the largest producer followed by Maharashtra, Karnataka,
Uttar Pradesh and Rajasthan. In terms of commodities oil seeds are the single largest category followed by
sugar crops, cereals and millets, fiber crops, pulses, medicinal, herbal and aromatic plants and spices and
condiments.
 The total volume of export during 2017-18 was 4.58 lakh tonnes. The organic food export realization was
around INR 3453.48 crore (515.44 million USD). Organic products are exported to USA, European
Union, Canada, Switzerland, Australia, Israel, South Korea, Vietnam, New Zealand, Japan etc. In terms
of export value realization oilseeds (47.6%) lead among the products followed by cereals and millets
(10.4%), plantation crop products such as tea and coffee (8.96%), dry fruits (8.88%), spices and
condiments (7.76%) and others.

Q 40.A
 Statement 1 is correct: „Amar Sonar Bangla‟, the national anthem of present-day Bangladesh, was
composed by Rabindranath Tagore during Swadeshi movement. It was sung by huge crowds marching in
the streets.
15 www.visionias.in ©Vision IAS
Join Our Telegram Channel https://t.me/UPSCMaterials For Instant Updates

 Statement 2 is not correct: Rabindranath Tagore renounced his knighthood to express the pain and
anger of the country with regard to the Jallianwala Bagh atrocities inflicted by General Dyer in
Amritsar on Baishakhi day.
 Statement 3 is not correct: The Cult of the Charkha is an essay by Rabindranath Tagore which first
appeared in September 1925 in the Modern Review. In the essay Tagore offered critique on the
Gandhian ethic of "charkha-spinning". Tagore remained unconvinced of the merit of Gandhi's forceful
advocacy that everyone should spin at home with the "charka," the primitive spinning wheel. For Gandhi
this practice was an important part of India's self-realization. The spinning-wheel gradually became the
center of rural uplift in the Gandhian scheme of Indian economics. However Tagore found the alleged
economic rationale for this scheme quite unrealistic.

Q 41.A
 It is the ministry of Home affairs that regulates Foreign Contribution Regulation Act
(FCRA) which deals with the regulation of foreign donations coming to several entities including
political parties, journalists etc. The focus of this Act is to make sure that the foreign contribution and
foreign hospitality are not used to influence electoral politics, public servants, judges and individuals
working in the important locations of nationwide life such as reporters, printers and publishers of
newspapers and so on.
 The Foreign Exchange Management Act (1999) or in short FEMA has been introduced as a replacement
for earlier Foreign Exchange Regulation Act (FERA). FEMA came into act on the 1st day of June, 2000.
The main objective behind the Foreign Exchange Management Act (1999) is to consolidate and amend the
law relating to foreign exchange with objective of facilitating external trade and payments and for
promoting the orderly development and maintenance of foreign exchange market in India. FEMA is
applicable to the all parts of India. The act is also applicable to all branches, offices and agencies outside
India owned or controlled by a person who is resident of India. Ministry of Finance is the authority for
the enforcement of FEMA.

Q 42.D
 A censure means an expression of strong disapproval or harsh criticism. It can be a stern rebuke by a
legislature, generally opposition against the policies of Government or an individual minister.
However, it can also be passed to criticise, condemn some act.
 A censure motion can be moved in lower house of the parliament or in a state assembly in
India. Usually we come across this word in context with the parliament of India, where it is moved by
opposition against a specific policy of government or against a minister or against the whole council of
ministers. If the censure motion is passed, the government does not need to resign, unlike in case of No-
confidence motion. Speaker's permission is not required to move a Censure Motion, unlike the No-
Confidence motion.

Q 43.A
 Statement 1 is correct: The Court is the principal judicial organ of the United Nations. It was
established by the United Nations Charter, which was signed in 1945 in San Francisco (United States),
and began work in 1946 in the Peace Palace, The Hague (Netherlands). The Court, which is composed of
15 judges, has a two-fold role: first, to settle, in accordance with international law, legal disputes between
States submitted to it by them and, second, to give advisory opinions on legal matters referred to it by
duly authorized United Nations organs and specialized agencies. The Court‘s official languages are
English and French.
 Statement 2 is correct: Only States are eligible to appear before the Court in contentious cases. The
Court has no jurisdiction to deal with applications from individuals, non-governmental organizations,
corporations or any other private entity. It cannot provide them with legal advice or help them in their
dealings with national authorities. The Court can only hear a dispute when requested to do so by one or
more States. It cannot deal with a dispute on its own initiative. Neither is it permitted, under its Statute, to
investigate and rule on acts of sovereign States as it chooses. Judgments delivered by the Court (or by one
of its Chambers) in disputes between States are binding upon the parties concerned.

16 www.visionias.in ©Vision IAS


Join Our Telegram Channel https://t.me/UPSCMaterials For Instant Updates

 Statement 3 is not correct: The International Court of Justice has no jurisdiction to try individuals
accused of war crimes or crimes against humanity. As it is not a criminal court, it does not have a
prosecutor who is able to initiate proceedings.

Q 44.B
 Kakatiyas (950 CE - 1323 CE): The sub-feudatories of the Rashtrakutas emerged themselves as
independent kings and founded the Kakatiya dynasty around 950 AD and this kingdom became strong and
united whole of Telugu-speaking lands and lasted for more than three centuries and a half. The kingdom
saw powerful kings like Ganapatideva, Rudradeva and Prataparudra as well as the first ever woman
ruler in the subcontinent Rudramadevi. The Kakatiyas ruled from Hanumakonda in the beginning
and shifted their capital to Warangal later.
 The Kakatiyas are known for their irrigation public works, sculpture and fire arts. Thanks to the well-
planned irrigation facilities and a perfect system of chain tanks to suit the undulating nature of the terrain,
the Kakatiya kingdom flourished economically leading to cultural progress also. Many Hindu temples
were developed under the patronage of Ganapati Deva, Rudrama Devi and Prataparudra who were of
Kakatiya dynasty. The Thousand Pillar Temple is believed to be constructed during the period between
1175–1324 CE by order of the king, Rudra Deva. It stands out to be a masterpiece and achieved major
heights in terms of architectural skills by the ancient Kakatiya vishwakarma sthapathis.
 Envy of this affluence, several neighbouring kingdoms as well as Delhi Sultanate tried to wage war on
Warangal many times and failed. Finally in 1323, Delhi army could lay seize on Warangal fort and
capture Prataparudra, who, according to the legend, killed himself on the banks of the Narmada unwilling
to surrender when he was being taken as prisoner of war to Delhi.

Q 45.B
 The government in a recent notification has declared printing presses and mints as public utility service
under the Industrial Disputes Act, 1947. The notification issued by the Ministry of Labour and
Employment is valid for six months and can be reissued after expiry.
 As per the The Industrial Disputes Act, 1947: Certain industries will be declared to be Public Utility
Services under the Act by the Government. In case of strike or lockout in respect of industries declared as
Public Utility Services, prior notice is compulsory either by employees or by management,
respectively. 39 industries are declared as PUS and included in the First Schedule to the Act. IT Software
Establishments and 100% EOU (Export Oriented Unit) are declared as PUS for a period of 2 years.
 THE FIRST SCHEDULE : Industries which may be Declared to be Public Utility Services Under
Sub-Clause (VI) of Clause (N) of Section 2
1. Transport (other than railways) for the carriage of passengers or goods, [by land or water].
2. Banking.
3. Cement.
4. Coal.
5. Cotton textiles
6. Foodstuffs
7. Iron and Steel.
8. Defense establishments.
9. Service in hospitals and dispensaries.
10. Fire Brigade Service.
11. India Government Mints.
12. India Security Press.
13. Copper Mining.
14. Lead Mining.
15. Zinc Mining.
16. Iron Ore Mining.
17. Service in any oilfield.
18. Service in the Uranium Industry.
19. Pyrites Mining Industry.
20. Security Paper Mill, Hoshangabad.

17 www.visionias.in ©Vision IAS


Join Our Telegram Channel https://t.me/UPSCMaterials For Instant Updates

21. Service in the Bank Note Press, Dewas.


22. Phosphorite Mining.
23. Magnesite Mining.
24. Currency Note Press.
25. Manufacture or production of mineral oil (crude oil), motor and aviation spirit, diesel oil, kerosene
oil, fuel oil, diverse hydrocarbon oils and their blends including synthetic fuels, lubricating oils and
the like.
26. Service in the International Airports Authority of India.
27. Industrial establishment, manufacturing or producing nuclear fuel and components, heavy water and
allied chemicals, and atomic energy.
 Hence, options 1, 2, 3 and 4 are correct and option 5 is not correct.

Q 46.C
 In a major step towards building a hack-proof global quantum communication network, China has sent an
unbreakable code from its quantum satellite to the Earth, marking realization of quantum key distribution
technology for the first time. China launched the world‟s first quantum satellite, nicknamed
“Micius” after a 5th Century Chinese philosopher and scientist, on August 16, 2016. Quantum key
technology is used in quantum communications to make eavesdropping impossible and to perfectly
secure the communication. Hence statement 3 is correct.
 Quantum communication takes advantage of the laws of quantum physics to protect data. These laws
allow particles—typically photons of light for transmitting data along optical cables—to take on a state
of superposition, which means they can represent multiple combinations of 1 and 0
simultaneously. The particles are known as quantum bits, or qubits. The beauty of qubits from a
cybersecurity perspective is that if a hacker tries to observe them in transit, their super-fragile quantum
state ―collapses‖ to either 1 or 0. This means a hacker can‟t tamper with the qubits without leaving
behind a telltale sign of the activity. Hence statement 1 is correct and statement 2 is not correct.

Q 47.D
 Statement 1 is not correct: Government commemorated the centenary of Champaran Satyagraha during
April 2017 to April 2018. Development of Gandhi circuit is part of this celebration.
 Statements 2 and 3 are not correct: The project is sanctioned under ―Rural Circuit theme of Swadesh
Darshan Scheme‖ with central financial assistance. The circuit covers the rural areas of Bihar having links
to Mahatma Gandhi. Development of Gandhi Circuit in Bihar is one of the projects identified for
development under the Special Package for Bihar announced by the Prime Minister in 2015. A project for
―Development of Gandhi Circuit: Bhitiharwa-Chandrahia-Turkaulia under Rural Circuit theme of
Swadesh Darshan Scheme‖ has been sanctioned in Bihar with central financial assistance.

Q 48.C
 "Progressive Realisation of Rights" doctrine was given by Supreme Court in one of its judgement which
decriminalised homosexuality. The five-judge Constitution Bench led by Chief Justice of India Dipak
Misra has introduced the “Doctrine of Progressive Realisation of Rights” to guard against future
attempts to reintroduce that part of Section 377 of the Indian Penal Code which had made the
entire LGBTQ community “unconvicted felons” without the basic rights of a citizen for over a
century. Using this legal doctrine, Chief Justice Misra has held that once a right is recognised and
given to the public, it cannot be taken back by the state at a later date. Once a step is taken forward,
there is no going back. ―In a progressive and an ever-improving society, there is no place for retreat or
regression. The society has to march ahead,‖ Chief Justice Misra held.

Q 49.A
 Statement 1 is not correct: The FRA recognises both individual forest rights and community forest
rights. This Act not only recognizes the rights to hold and live in the forest land under the individual or
common occupation for habitation or for self-cultivation for livelihood, but also grants several other rights
to ensure their control over forest resources which, inter-alia, include right of ownership, access to collect,
use and dispose of minor forest produce, community rights such as nistar; habitat rights for primitive
18 www.visionias.in ©Vision IAS
Join Our Telegram Channel https://t.me/UPSCMaterials For Instant Updates

tribal groups and pre-agricultural communities; right to protect, regenerate or conserve or manage any
community forest resource which they have been traditionally protecting and conserving for sustainable
use.
 Statement 2 is correct: The Act also provides for diversion of forest land for public utility facilities
managed by the Government, such as schools, dispensaries, fair price shops, electricity and
telecommunication lines, water tanks, etc. with the recommendation of Gram Sabhas.
 Statement 3 is not correct: The recognition and vesting of forest rights under this Act to the forest
dwelling Scheduled Tribes and to other traditional forest dwellers in relation to any State or Union
territory in respect of forest land and their habitat is subject to the condition that such Scheduled Tribes or
tribal communities or other traditional forest dwellers had occupied forest land before the 13th day of
December, 2005.

Q 50.D
 Fly ash or flue ash, also known as pulverised fuel ash, is a coal combustion product that is composed of
the particulates (fine particles of burned fuel) that are driven out of coal-fired boilers together with the
flue gases.
 Fly ash uses include:
o Concrete production, as a substitute material for Portland cement,sand.
o Fly-ash pellets which can replace normal aggregate in concrete mixture.
o Embankments and other structural fills (usually for road construction)
o Grout and Flowable fill production
o Waste stabilization and solidification
o Cement clinker production - (as a substitute material for clay)
o Mine reclamation
o Stabilization of soft soils
o Road subbase construction
o As aggregate substitute material (e.g. for brick production)
o Mineral filler in asphaltic concrete
o Agricultural uses: soil amendment, fertilizer, cattle feeders, soil stabilization in stock feed yards, and
agricultural stakes
o Loose application on rivers to melt ice.
o Loose application on roads and parking lots for ice control.

Q 51.D
 The houses of common people differed in size from a single-room house to bigger structures. The
houses were largely built of burnt bricks. The bigger houses had many rooms surrounding a square
courtyard. These houses were provided with private wells, kitchens and bathing platforms. The difference
in the size of the houses suggests that the rich lived in the larger houses whereas the one-room buildings
or barracks might have been intended for the poorer section of the society. Hence statement 1 is not
correct.
 At Mohenjodaro, the „Great Bath‟ is the most important structure. It is surrounded by corridors on all
sides and is approached at either end by steps in north and south. A thin layer of bitumen was applied to
the bed of the Bath to ensure that water did not seep in. Water was supplied by a large well in an adjacent
room. There was a drain for the outlet of the water. The bath was surrounded by sets of rooms on sides for
changing cloth. Scholars believe that the ‗Great Bath‘ was used for ritual bathing. The dominant sight
at Lothal is the massive tank. Scholars believe that this tank may have been a dockyard, where boats
and ships came in from the sea and through the river channel. Goods were probably loaded and unloaded
here. So, archaeological sources found Great Bath in Mohenjodaro and dockyard in Lothal. Hence
statement 2 is not correct.
 Though there is no evidence of temples in Harappan cities, fire altars at Kalibangan and Lothal
suggest that sacrifices may have been performed at these altars. Hence statement 3 is not correct.
19 www.visionias.in ©Vision IAS
Join Our Telegram Channel https://t.me/UPSCMaterials For Instant Updates

Q 52.D
 Fiscal Drag means an adverse effect of progressive taxation on demand and economic expansion.
 It occurs when due to the rise in national income, it brings in more and more people under the taxation net
due to progressive taxation and due to rise in their incomes. However, while the nominal income has gone
up, but real income may not have correspondingly increased, as the higher taxation leads to reduction in
aggregate demand which may bring in slowdown in the economy due to low demand.
 It is basically automatic restraint applied to a fast expanding economy under a progressive tax system. As
incomes rise, individuals are pushed into higher income tax brackets, and average tax obligations increase
that dampen spending.

Q 53.A
 Jerdon‟s Courser (Rhinoptilus bitorquatus) is a nocturnal bird endemic to the Eastern Ghats of
Andhra Pradesh and extreme southern Madhya Pradesh. It is one of the world‘s rarest bird species
and is classified as Critically Endangered (CR) by the International Union for Conservation of
Nature (IUCN). The species was believed to be extinct until it was rediscovered in 1986 near Reddipalli
village, Cuddapah District of Andhra Pradesh, India. The site where it was rediscovered was designated
as the Sri Lankamaleswara Wildlife Sanctuary.

Q 54.D
 Rajkot Satyagraha was started against the breach of the charter of Liberty of the people by the local
ruler Thakore Saheb instigated by the British Resident in Rajkot. Lakhajiraj, a ruler of Rajkot Princely
State died in 1939 and his son Dharmendra Singhji, a complete contrast to the father, soon took charge of
the State. The new Thakore was interested only in pleasure, and effective power fell into the hands of
Dewan Virawala. Soon, the State finances reached such a pass that the State began to sell monopolies for
the sale of matches, sugar, rice, and cinema licences to individual merchants. This immediately resulted in
a rise in prices and enhanced the discontent that had already emerged over the Thakore‘s easy-going life-
style.
 On 7 March 1939 Gandhiji broke his fast after Viceroy asked Chief Justice of India, Sir Maurice
Gwyer, to decide whether in fact the Thakore had violated the agreement. This seemed a reasonable
enough proposition, and Gandhiji broke his fast. Later Gandhiji, analyzing the reasons for his failure to
achieve a ‗change of heart‘ in his opponents, came to the conclusion that the cause lay in his attempt to
use the authority of the Paramount Power to coerce the Thakore into an agreement. This, for him,
smacked of violence; non-violence should have meant that he should have directed his fast only at the
Thakore and Virawala, and relied only on the strength of his suffering to effect a ‗change of heart‘.
Therefore, he released the Thakore from the agreement, apologized to the Viceroy and the Chief Justice
for wasting their time, and to his opponents, the Muslims and the Rajputs, and left Rajkot to return to
British India.

Q 55.D
 The Preamble to the Constitution of India reads,
 ―We, THE PEOPLE OF INDIA, having solemnly resolved to constitute India into a SOVEREIGN
SOCIALIST SECULAR DEMOCRATIC REPUBLIC and to secure to all its citizens: JUSTICE, Social,
Economic and Political; LIBERTY of thought, expression, belief, faith and worship; EQUALITY of
status and of opportunity; and to promote among themall; FRATERNITY assuring the dignity of the
individual and the unity and integrity of the Nation; IN OUR CONSTITUENT ASSEMBLY this twenty-
sixth day of November, 1949, do HEREBY ADOPT, ENACT AND GIVE TO OURSELVES THIS
CONSTITUTION‖. Hence options 2 and 3 are correct.
 Although the word 'Federation' has nowhere been mentioned in the Constitution, the Constitution
establishes a federal system of government which is evident through the following:- 1. Written
Constitution 2. Supremacy of the Constitution 3. Government at two levels 4. Independent Judiciary 5.
Bicameralism 6.Division of Powers (Existence of three lists in the Constitution Union List, State List and
the Concurrent List). Hence, option 1 is not correct.
 The Constitution of India provides for a parliamentary form of government, but the parliamentary system
is not mentioned in the Preamble to the Indian Constitution. Hence, option 4 is not correct.
20 www.visionias.in ©Vision IAS
Join Our Telegram Channel https://t.me/UPSCMaterials For Instant Updates

Q 56.A
 Macaulay duration, named after Frederick Macaulay who developed the concept, is a measure of
how long it takes for the price of a bond to be repaid by the cash flows from it. It is the weighted
average number of years an investor must maintain a position in the bond until the present value of the
bond's cash flows equals the amount paid for the bond.

Q 57.D
 Statement 1 is not correct and statement 2 is correct: India has 15,106.7 km of land border and a
coastline of 7,516.6 km including island territories. India shares the longest border with Bangladesh
(4096.7 km).

 India shares a 3,323 km border with Pakistan. The border is divided into three parts:
o First is the International Border (IB), which stretches for approximately 2,400 km from Gujarat to the
north banks of Chenab in Akhnoor in Jammu.
o Second is the Line of Control (LoC), which is 740 km long and runs from parts of Jammu to parts of
Leh. It is a ceasefire line which came into existence after the 1948 and 1971 wars between India and
Pakistan. It was delineated in the Simla Agreement (July 1972) whereby both sides agreed not to
alter it unilaterally.
o Third is the Actual Ground Position Line (AGPL), which divides the current position of Indian and
Pakistani troops in the Siachen region. It is 110 km long and extends from NJ 9842 to Indira Col in
the north.
 Statement 3 is correct: At the 1914 China-Tibet-Britain Simla Convention, Sir Henry McMahon, foreign
secretary of British India, drew up the 890-km McMahon Line as the border between British India
and Tibet. The line, drawn primarily on the highest watershed principle, marked out previously
unclaimed/undefined borders between Britain and Tibet. The line put Tawang and other Tibetan areas in
the British empire. The line was forgotten until the British government published the documents in the
1937 edition of Aitchison‘s Collection of Treaties. Subsequently, China refused to accept the line.

Q 58.A
 The Paik system was a type of corvee labor system on which the economy of the Ahom kingdom of
medieval Assam depended, and it is believed that the system is based on the South-East Asian
legacy the Ahoms brought with them in 1228.In this system, adult and able males were obligated to
render service to the state and form its militia, and in return for which they received a piece of land for
cultivation. The mature structure was designed by Momai Tamuli Borbarua in 1608, and extensively and
21 www.visionias.in ©Vision IAS
Join Our Telegram Channel https://t.me/UPSCMaterials For Instant Updates

exhaustively implemented by 1658 during the reign of Sutamla Jayadhwaj Singha. The system continued
to evolve over time to meet the needs of the Ahom state and in time began to accumulate contradictions.
By the end of the Moamoria rebellion (1769–1805) the Paik system had collapsed.

Q 59.B
 Statement 1 is not correct: Even if a state law prescribes for death sentence, the power to grant pardon
lies with the President and not the governor. But, the governor can suspend, remit or commute a death
sentence. The President is the only authority to pardon a death sentence. Governor cannot pardon a death
sentence.
 Statement 2 is correct:The President can grant pardon in respect to punishment or sentence by a court-
martial (military court), while Governor's office does not have any power in regards with military courts.

Q 60.B
 Fiscal Responsibility and Budget Management (FRBM) became an Act in 2003. The objective of the Act
is to ensure inter-generational equity in fiscal management, long run macroeconomic stability, better
coordination between fiscal and monetary policy, and transparency in fiscal operation of the Government.
 It requires for the presentation of the following documents before the Parliament - the Medium Term
Expenditure Framework Statement (MTEF), Medium-Term Fiscal Policy Statement, Fiscal Policy
Strategy Statement and Macroeconomic Framework Statement.
 The introduction of Outcome Budget is an executive action by the government. From the year 2017-18
onwards, it has been decided that the output and outcomes of the schemes of 68 Ministries and
Departments will be available along with the financial outlays as a part of the Budget documents, so that
clearly defined objectives and goals for each scheme can be seen by all.
Hence, option (b) is the correct answer.

Q 61.D
 Astana is capital of Kazakhstan. Astana lies in the north-central part of the country, along the Ishim River,
at the junction of the Trans-Kazakhstan and South Siberian railways.
 Tashkent is capital of Uzbekistan and the largest city in Central Asia. Tashkent lies in the northeastern
part of the country.
 Dushanbe is capital of Tajikistan. It lies along the Varzob (Dushanbinka) River in the Gissar valley, in the
southwest of the republic.
 Kabul is capital of the province of Kabul and of Afghanistan. The largest urban centre in the country and
also its political and economic hub, the city forms one of several districts of Kabul province.

22 www.visionias.in ©Vision IAS


Join Our Telegram Channel https://t.me/UPSCMaterials For Instant Updates

Q 62.B
 Kanishka was the most important ruler of the Kushana dynasty. At the time of his accession his empire
included Afghanistan, Gandhara, Sind and Punjab. The empire of Kanishka was a vast one extending
from Gandhara in the west to Benares in the east, and from Kashmir in the north to Malwa in the
south. His capital was Purushapura or modern day Peshawar.
 Though Kanishka was a Buddhist, coins from that era, the Saka era, depict religious deities of Greek,
Sumerian, Zoroastrian and Indian origin. It reflects the Kanishka‘s toleration towards other
religions.
 He patronised Buddhist scholars like Vasumitra, Asvagosha and Nagarjuna. He also convened the Fourth
Buddhist Council to discuss matters relating to Buddhist theology and doctrine. It was held at the
Kundalavana monastery near Srinagar in Kashmir under the presidentship of Vasumitra. About
500 monks attended the Council. The Council prepared an authoritative commentary on the Tripitakas and
the Mahayana doctrine was given final shape. Asvagosha was a great philosopher, poet and dramatist. He
was the author of Buddhacharita. Nagarjuna from south India adorned the court of Kanishka. The
famous physician of ancient India Charaka was also patronized by him.
 During Kushana rule kings adopted grandiloquent titles.They bore titles such as maharaja, devaputra (son
of God), etc. Kanishka and his successors used the title shaonano shao (shahanushahi being its Persianized
form) as a prefix to their names on the coin legends. Kanishka attempted to attach divine status to the
king by assuming the title devaputra.

Q 63.A
 Edge Computing is a mesh network of microdata centres that process or store critical records locally
and push all received records to a central data centre or cloud storage repository, in a footprint of
less than 100 square feet.
 Hence, as opposed to the Internet of Things which collects a loads of data and sends it directly to a cloud,
edge computing actually processes this data within a network. Consequently, only the relevant data
or information is collected and then sent, rather than all of the data collected.
 This helps to reduce latency issues, and also reduces the amount of irrelevant data stored. Thus, it helps in
lowering costs, more security, more faster, more dependable and works on multiple devices.
 Hence, option (a) is the correct answer.

Q 64.A
 The four-fold classification Buddhist philosophy is based on two major philosophical problems. The first
is the metaphysical question: Is there any reality, mental or non mental? One school replies that reality is
neither mental or non mental, that all is Sunya. This school is known as Sunyavada. According to another
school, reality is only mental. This school is known as Vijnanavada. Hence statement 1 is not correct.
 The founder of Sunyavada school is said to be Nagarjuna. According to this school, all that is perceived
within or without is unreal, like the objects of dream. There is nothing mental or non mental that is
real. The entire universe is unreal. Hence statement 2 is correct.
 Sunyavada also called as Madhyamika is often viewed as the basic school of Mahayana Buddhism. The
authoritative texts of the school are the Madhyamikasastra by Nagarjuna and the Satakasastra
attributed to his pupil Aryadeva. These texts were written in Sanksrit. Hence statement 3 is not
correct.

Q 65.B
 Statement 1 is correct: Exchange rates in the market depend not only on the demand and supply of
exports and imports, and investment in assets, but also on foreign exchange speculation where foreign
exchange is demanded for the possible gains from appreciation of the currency. Money in any country is
an asset. If Indians believe that the British pound is going to increase in value relative to the rupee, they
will want to hold pounds. For instance, if the current exchange rate is Rs 80 to a pound and investors
believe that the pound is going to appreciate by the end of the month and will be worth Rs 85, investors
think if they took Rs 80,000 and bought 1,000 pounds, at the end of the month, they would be able to

23 www.visionias.in ©Vision IAS


Join Our Telegram Channel https://t.me/UPSCMaterials For Instant Updates

exchange the pounds for Rs 85,000, thus making a profit of Rs 5,000. This expectation would increase the
demand for pounds and cause the rupee-pound exchange rate to increase in the present, making the beliefs
self-fulfilling.
 Statement 2 is not correct: A rise in the interest rates of bonds at home often leads to an appreciation of
the domestic currency. If we assume that government bonds in country A pay 8 per cent rate of interest
whereas equally safe bonds in country B yield 10 per cent, the interest rate differential is 2 per cent.
Investors from country A will be attracted by the high interest rates in country B and will buy the currency
of country B selling their own currency. At the same time investors in country B will also find investing in
their own country more attractive and will therefore demand less of country A's currency. This means that
the demand curve for country A‘s currency will shift to the left and the supply curve will shift to the right
causing a depreciation of country A‘s currency and an appreciation of country B‘s currency.
 Statement 3 is correct: A surplus of exports over imports for India will cause an increase in demand for
Indian rupee (overseas buyers need the Indian rupee to pay for the goods) and will therefore lead to
appreciation of Indian rupee.
 However, a deficit situation in which Indian imports exceed exports (a trade deficit) will cause an increase
in supply of the foreign currency required to pay for the imports and will therefore lead to depreciation of
Indian rupee.

Q 66.C
 Statement 1 is not correct: Indo-Islamic architecture begins with the Ghurid occupation of India at the
close of the 12 century A.D. The rule of Mughal kingdom in India started in 16th century.
 Statement 2 is correct: One of the major buildings of Jahangir's period is the tomb of Akbar. Located in
Sikandra, the tomb is situated in the middle of garden. The tomb is a square structure built in three stories.
The decorative motifs used in the tomb include traditional floral designs, arabesques and calligraphy, the
elephant, the lotus, swastika and charka.
 Statement 3 is correct: Buland Darwaza or the loft gateway at Fatehpur Sikri was built by the great
Mughal emperor, Akbar in 1601. Akbar built the Buland Darwaza to commemorate his victory over
Gujarat. The Buland Darwaza is the highest gateway in the world and an astounding example of the
Mughal architecture. It is made of red and buff sandstone, and decorated by carving and inlaying of white
and black marble. An inscription on the central face of the Buland Darwaza throws light on Akbar's
religious tolerance and broad mindedness.

Q 67.D
 Statement 1 is not correct. Under proxy voting, a registered elector can delegate his voting power to a
representative.This was introduced in 2003 for elections to the Lok Sabha and Assemblies, but on a
limited scale. The proxy shall have to be ordinary resident of that constituency. They need not be
registered voters, but he / she must not be disqualified to be registered as a voter.
 Statement 2 is not correct. Only classified service voters are allowed voting through Proxy voting, not
the Overseas Citizens of India. Service voter is a voter having service qualification. According to the
provisions of sub – section (8) of Section 20 of Representation of People Act, 1950, service qualification
means –
o Being a member of the armed Forces of the Union ; or
o Being a member of a force to which provisions of the Army Act, 1950 (46 of 1950), have been made
applicable whether with or without modification ;
o Being a member of an Armed Police Force of a State, and serving outside that state; or
o Being a person who is employed under the Government of India, in a post outside India.

Q 68.D
 Statement 1 is correct: India receives 70-80 percent of rainfall from South west monsoon. Southwest
monsoon normally sets in over Kerala around 1st June. It advances northwards, usually in surges, and
covers the entire country around 15th July.
 Statement 2 is correct: Two significant features of India's rainfall are in the north India, rainfall
decreases westwards and in Peninsular India, except Tamil Nadu, it decreases eastward. The highest
rainfall occurs in west coast as the Sub-Himalayan areas in North East and Meghalaya Hills, Assam and

24 www.visionias.in ©Vision IAS


Join Our Telegram Channel https://t.me/UPSCMaterials For Instant Updates

West Bengal Himalayas (Over 200cm) Moderate rainfall (100-200 cm) occurs in Southern parts of
Gujarat, East Tamil Nadu. Upper Ganga valley, eastern Rajasthan, Punjab, Southern Plateau of Karnataka,
Andhra Pradesh experience less Rainfall (50-100 cm). Areas of Scanty Rainfall (Less than 50 cm):
Northern part of Kashmir, Western Rajasthan, Punjab and Deccan Plateau get scanty rainfall.
 Statement 3 is correct: LPA is the average rainfall received by the country as a whole during the south-
west monsoon, for a 50-year period. The current LPA is 89 cm, based on the average rainfall over years
1951 and 2000. This acts as a benchmark against which the rainfall in any monsoon season is measured.
The country is said to have received deficient rainfall if the actual rainfall falls below 90 per cent of LPA.
Similarly, the country is said to have received excess rainfall if the rainfall is greater than 110 per cent of
LPA.

Q 69.A
 SWAYATT (Startups, Women and Youth Advantage Through eTransactions) is an initiative by
Ministry of Commerce & Industry to promote Start-ups through eTransactions on Government e
Marketplace (GeM). This will bring together the key stakeholders within the Indian entrepreneurial
ecosystem to Government e-Marketplace the national procurement portal.
 It seeks to promote inclusiveness by catapulting various categories of sellers and service providers, take
proactive steps to facilitate the training and registrations of such specific category of manufacturers and
sellers, develop women entrepreneurship and encourage participation of MSME sector and Start-ups in
public procurement.

Q 70.C
 Statement 1 is not correct: Ozone depletion is the gradual thinning of Earth‘s ozone layer in the upper
atmosphere (Stratospheric region) caused by the release of chemical compounds containing gaseous
chlorine or bromine from industry and other human activities.
 Statement 2 is not correct: This condition does not lead to ozone depletion. In the summer season,
nitrogen dioxide and methane react with chlorine monoxide and chlorine atoms forming chlorine sinks,
preventing much ozone depletion.
 Statement 3 is correct: Extra ultraviolet B radiation reaching Earth also inhibits the reproductive
cycle of phytoplankton, single-celled organisms such as algae that make up the bottom rung of the food
chain. Biologists fear that reductions in phytoplankton populations will in turn lower the populations of
other animals. The thinning is most pronounced in the polar regions, especially over Antarctica. Ozone
depletion is a major environmental problem because it increases the amount of ultraviolet (UV) radiation
that reaches Earth‘s surface, which increases the rate of skin cancer, eye cataracts, and genetic and
immune system damage.

Q 71.A
 The Reserve Bank of India gives temporary loan facilities to the centre and state governments as a
banker to government. This temporary loan facility is called Ways and Means Advances (WMA).
 The WMA scheme for the Central Government was introduced on April 1, 1997, after putting an end to
the four-decade old system of adhoc (temporary) Treasury Bills to finance the Central Government
deficit. The WMA scheme was designed to meet temporary mismatches in the receipts and payments of
the government. This facility can be availed by the government if it needs immediate cash from the RBI.
The WMA is to be vacated after 90 days.
 Under the WMA scheme for the State Governments, there are two types of WMA – Special and Normal
WMA. Special WMA is extended against the collateral (mortgaging) of the government securities held by
the State Government. After the exhaustion of the special WMA limit, the State Government is provided a
normal WMA. The normal WMA limits are based on three-year average of actual revenue and capital
expenditure of the state.

Q 72.C
 Like any other written Constitution, the Constitution of India also provides for its amendment in order to
adjust itself to the changing conditions and needs.

25 www.visionias.in ©Vision IAS


Join Our Telegram Channel https://t.me/UPSCMaterials For Instant Updates

 Statement 1 is not correct: An amendment of the Constitution can be initiated only by the introduction
of a Bill for the purpose in either House of Parliament, not necessarily Lok Sabha. But it can not be
introduced in the State Legislatures.
 Statement 2 is not correct: Introduction of a Constitutional Amendment bill under Atricle 368 does not
require prior permission of the President. But, in certain cases, when alteration to any part of the
Constitutions does not come under Article 368, prior permission of the President is required. Such a bill is
often treated as an ordinary bill.
 Statement 3 is correct: When a Constitutional Amendment bill is presented to the President for assent
President must give his assent to the bill. He can neither withhold his assent to the bill nor return the bill
for reconsideration of the Parliament.

Q 73.D
 The STAR is a short name of the System for Transparent Allocation of Resources.
 The STAR determines the amount of GEF resources that a given country can access in a
replenishment period
 With the STAR, the GEF Secretariat allocates resources in an indicative way to its eligible countries in a
replenishment period.
 It was developed during 2009-2010 to upgrade the Resource Allocation Framework (RAF), which was the
former GEF resource allocation system used in the fourth replenishment period of the GEF (GEF-4).
 In the fifth replenishment period of the GEF (GEF-5), the STAR covers three focal areas:
o Biodiversity (BD)
o Climate change (CC)
o Land degradation (LD).
 Other focal areas and programs may be covered by the STAR in the future GEF replenishment periods

Q 74.A
 Kailasanath Temple: The Kailasanathar temple at Kanchipuram was built by Raja Simha shortly after
the Shore temple in the 8th century A.D., and compared to the latter, is larger in dimensions and more
majestic in appearance. The Kailashnath temple is situated in a rectangular courtyard surrounded by a
peristyle composed of a continuous series of cells resembling rathas. It consists of the sanctum (garbha
griha), a pillared hall (mandapa), the ambulatory, the vestibule in the shape of a hall. The flat-roofed
pillared mandapa, which was a separate building originally, was connected with the sanctum by a
vestibule.
 Konark Sun Temple: Sun temple at Konark which was constructed by the eastern Ganga ruler
Narasimha Varmana, about 1250 A.D. It is a vast and wonderful structure, magnificently conceived as a
gigantic chariot with 12 pairs of ornamental wheels, pulled by seven rearing horses. The colossal temple
originally consisted of a sanctum sanctorum, with a lofty curvilinear shikhara, a Jagamohana and a
dancing hall, built on the same axis, and an extensive compound wall with three entrance gateways.
 Dilwara Temples: Wondrously carved in marble, the Dilwara Jain temples in Mount Abu (Rajasthan)
enshrine various Jain 'Tirthankaras'. Constructed out of white marble these temples are an outstanding
example of Jain temple architecture. Of the five shrines in this group, four are architecturally significant.
Each has a walled courtyard. In the centre of the courtyard is the shrine with the image of the deity,
Rishabhdev. Around the large courtyard, there are numerous small shrines, each housing a beautiful
image of the tirthankaras with a series of elegantly carved pillars from the entrance to the courtyard. The
ministers of the Solanki rulers of Gujarat had constructed all these temples during 11th and 13th
centuries A.D. Vimal Vasahi is the oldest temple, which has been dedicated to Adinath, the first Jain
tirthankara. Vimal Shah, minister of the then Solanki ruler of Gujarat, built it in the year 1031 A.D.
 Hazara Rama Temple: Hazara Rama temple in Hampi is an important shrine in Hampi. The temple is
famous for the lovely bas relics and panels depicting the story of the epic Ramayana. Its construction was
started in the year 1513 A.D., under the orders of Krishnadevaraya and was completed before the end of
his reign.

26 www.visionias.in ©Vision IAS


Join Our Telegram Channel https://t.me/UPSCMaterials For Instant Updates

Q 75.A
 The Protection of Human Rights Act of 1993 provides for the creation of not only the National Human
Rights Commission but also a State Human Rights Commission at the state level. Hence, statement 1 is
correct.
 A State Human Rights Commission can inquire into violation of human rights only in respect of subjects
mentioned in the State List (List-II) and the Concurrent List (List-III) of the Seventh Schedule of the
Constitution. However, if any such case is already being inquired into by the National Human Rights
Commission or any other Statutory Commission, then the State Human Rights Commission does not
inquire into that case.
 The State Human Rights Commission is a multi-member body consisting of a chairperson and two
members. The chairperson should be a retired Chief Justice of a High Court and members should be a
serving or retired judge of a High Court or a District Judge in the state with a minimum of seven years
experience as District Judge and a person having knowledge or practical experience with respect to human
rights. Hence, statement 2 is not correct.
 Although the chairperson and members of a State Human Rights Commission are appointed by the
governor, they can be removed only by the President (and not by the governor). Hence, statement 3
is correct.
 The President can remove them on the same grounds and in the same manner as he can remove the
chairperson or a member of the National Human Rights Commission. Thus, he can remove the
chairperson or a member under the following circumstances:
o If he is adjudged an insolvent; or
o If he engages, during his term of office, in any paid employment outside the duties of his office; or
o If he is unfit to continue in office by reason of infirmity of mind or body; or
o If he is of unsound mind and stands so declared by a competent court; or
o If he is convicted and sentenced to imprisonment for an offence.In addition to these, the president can
also remove the chairperson or a member on the ground of proved misbehaviour or incapacity.
However, in these cases, the President has to refer the matter to the Supreme Court for an inquiry. If
the Supreme Court, after the inquiry, upholds the cause of removal and advises so, then the President
can remove the chairperson or a member.

Q 76.A
 The Van Allen radiation belt is a torus of energetic charged particles (i.e. a plasma) around Earth,
trapped by Earth's magnetic field. The Van Allen belts are closely related to the polar aurora where
particles strike the upper atmosphere and fluoresce. They are shaped like Giant donut-shaped swaths of
magnetically trapped, highly energetic charged particles surrounding the Earth.
 There are two belts of asteroids in the solar system- one between mars and jupiter and other beyond
neptune.

Q 77.A
 POSHAN Abhiyaan (National Nutrition Mission - NNM) launched by Government in November 2017
aims to reduce the level of stunting, under-nutrition, anemia and low birth weight babies.
 Statement 1 is correct: Under POSHAN Abhiyaan, the Government has fixed targets to reduce stunting,
under-nutrition, anemia (among young children, women and adolescent girls) and reduce low birth weight
by 2%, 2%, 3% and 2% per annum respectively.
 Statement 2 is correct: All States and districts are covered. Coverage of districts in a phased manner is
315 districts in 2017-18, 235 districts in 2018-19 and remaining districts in 2019-20.
 Statement 3 is not correct: No cash assistance is given under POSHAN Abhiyaan. Whereas,
under Pradhan Mantri Matru Vandana Yojana (PMMVY), a cash incentive of ` 5000/- would be provided
directly in the account of Pregnant Women and Lactating Mothers (PW&LM) for first living child of the
family subject to their fulfilling specific conditions relating to Maternal and Child Health.
 The important components of the POSHAN Abhiyaan are:
o ensuring convergence with various programmes; incentivizing States/ UTs for achieving goals;
o IT-enabled Real-Time Monitoring (ICT-RTM); Evaluation; weighing efficiency and making nutrition
visible;

27 www.visionias.in ©Vision IAS


Join Our Telegram Channel https://t.me/UPSCMaterials For Instant Updates

o Community mobilization awareness advocacy;


o IEC, Jan Andolan - to educate the people on nutritional aspects, on-line Course on Nutrition for
Children, Nutrition message from folk songs and songs on WASH, sending messages on nutrition and
also creating ring-tones, Yoga for children at AWCs; strengthening human resource; measuring height
and length of children below 6 years of age.

Q 78.A
 Ecological succession is the process of change in the species structure of an ecological community over
time. The time scale can be decades (for example, after a wildfire), or even millions of years after a mass
extinction.
 Statement 1 is correct: During the process of succession, the population of organisms shift the nutrients
from their reservoirs. Example - Plants take up carbon dioxide from its reservoir (the atmosphere) and use
it to produce energy through photosynthesis.
 Statement 2 is not correct: There is an overall increase in the diversity of organisms from no organisms
prior to succession to only pioneer species at the start of it to finally a well-developed and diverse climax
community.
 Statement 3 is correct: As the process of succession occurs, there is an increased niche development
where the individual species adapt themselves to the ecosystem to reduce the competition among
themselves.
 Statement 4 is correct: The climax community which is the last stage of ecological succession is
characterized by a complex food web indicating interdependence among the species in the climax
community.

Q 79.B
 The Kaveri rises in the Brahmagri range of the Western Ghats and it reaches the Bay of Bengal in south
of Cuddalore, in Tamil Nadu. Total length of the river is about 760 km. Its main tributaries are Amravati,
Bhavani, Hemavati and Kabini. Its basin drains parts of Karnataka, Kerala and Tamil Nadu.
 The island town of Shivasamudram lies about 65 kms east of Mysore in Mandya district. Here the River
Kaveri meanders to fall into cascading Sivasamudram Falls. The falls are 893 metres wide.
 The Narmada originates on the western flank of the Amarkantak plateau at a height of about 1,057 m.
Flowing in a rift valley between the Satpura in the south and the Vindhyan range in the north, it forms a
picturesque gorge in marble rocks and Dhuandhar waterfall near Jabalpur.
 Dudhsagar Falls is located 10 km from Collem in Goa, in an evergreen valley in the Western Ghats. It
cascades down 306 mts in two segments. It is formed by the River Mandovi.
 One of the highest waterfalls in India, Jog Falls is one of the major attractions in Karnataka. Jog Falls is
also known by alternative names of Gersoppa Falls and Jogada Gundi. It is located in Shimoga district.
Jog Falls is formed by the Sharavathi River, falling from a height of 253 meters. The river originates at
Ambuthirtha in Thirthahalli taluk, flows north-west through the Western Ghats forming the Jog Falls
before joining the Arabian Sea at Honnavar.

Q 80.A
 Liaquat - Nehru Pact, a bilateral treaty between India and Pakistan was signed on April 8, 1950. It was
the outcome of six days of talks between India and Pakistan. The Prime Ministers of India and Pakistan,
Jawaharlal Nehru and Liaquat Ali Khan wanted to ensure the rights of minorities in both countries.
 Article 35A was brought in by presidential order in 1954 in order to safeguard the rights and guarantee
the unique identity of the people of Jammu&Kashmir. It is a constitutional provision that allows the
Jammu & Kashmir assembly to define permanent residents of the state. According to the
Jammu&Kashmir constitution, a Permanent Resident is defined as a person who was a state subject on
May 14, 1954, or who has been residing in the state for a period of 10 years, and has ―lawfully acquired
immovable property in the state‖. Only the Jammu & Kashmir assembly can change the definition of
Permanent Resident through a law ratified by a two-third majority.
 The privy purse was a monetary benefit provided to erstwhile princely states in return of their
sovereignty claim. It was abolished in 1971 as a 26th constitutional amendment. Then Prime Minister
Indira Gandhi argued the case for abolition based on equal rights for all citizens and the need to reduce the

28 www.visionias.in ©Vision IAS


Join Our Telegram Channel https://t.me/UPSCMaterials For Instant Updates

government's revenue deficit. Article 363-A was inserted in the Constitution (Twenty-sixth Amendment)
Act, 1971 for the abolition of Privy Purse.
 The first proclamation of National Emergency was issued in October 1962 on account of Chinese
aggression in the NEFA (North-East Frontier Agency now Arunachal Pradesh), and was in force till
January 1968. Hence, a fresh proclamation was not needed at the time of war against Pakistan in 1965.

Q 81.B
 Mysuru Dasara is the Nadahabba of the state of Karnataka in India. It is a 10-day festival, starting
with Navaratri and the last day being Vijayadashami. The festival is observed on the tenth day in the
Hindu calendar month of Ashvin, which typically falls in the Gregorian months of September and
October. The festivities begin by lighting the lamp and offering flowers to the idol of Goddess
Chamundeshwari. The festival is being celebrated since 1610 and is patronized by the Wodeyar royal
family.
 Gowri Habba or festival is celebrated a day before Ganesh Chaturthi. It is a significant festival in
parts of Karnataka, Andhra Pradesh & Tamil Nadu. Goddess Gowri, wife of Lord Shiva, the mother
of Lord Ganesha and Lord Subramanya is worshipped throughout India for her ability to bestow upon her
devotee‘s power, courage, valour. She is the most powerful of all the Goddess and is the very incarnation
of Aadhi Shakthi Mahamaya.
 Bathukamma is floral festival celebrated predominantly by the Hindu women of Telangana and
some parts of Andhra Pradesh. Every year this festival is celebrated as per Shathavahana calendar for
nine days starting Bhadrapada Pournami till Durgashtami, usually in September–October of
Gregorian calendar. For one whole week, women make small „Bathukammas‘, play around them
every evening and immerse them in a nearby water pond. On the last day, menfolk of the house go into
the wild plains and gather the flowers like ‗gunuka‘ and ‗tangedu‘. They bring home bagfuls of these
flowers and the entire household sits down to arrange them in stacks. The flowers are carefully arranged
row after row in a brass plate (called ‗taambalam‘) in circular rows and in alternate colours. The
Bathukamma grows in size and the bigger it gets the better.
 The Mahamastakabhisheka Mahotsava is an anointing ceremony of the statue of Lord Bahubali,
the son of lord Rishabhanatha who was the first of the 24 Jain Tirthankaras. Also referred as Grand
Consecration, Mahamastakabhisheka is performed once in 12 years. The 'padabhisheka' to the statue is
held every year. This year, Mahamastakabhisheka Mahotsava was observed in February 2019. The
sculpture of Lord Bahubali stands for renunciation, self-control and subjugation of the ego as a reflection
of his life.

Q 82.A
 Statement 1 is correct: During a national emergency, the Centre becomes entitled to give executive
directions to a state on „any‟ matter. Thus, the state governments are brought under the complete
control of the Centre, though they are not suspended.
 Statement 2 is not correct: Also, during a national emergency, the Parliament becomes empowered to
make laws on any subject mentioned in the State List. Although the legislative power of a state
legislature is not suspended, it becomes subject to the overriding power of the Parliament.
 Statement 3 is not correct: It is the Parliament, not the President, who may extend the life of Parliament
beyond its normal term (five years), by a law for one year at a time (for any length of time). Similarly, the
Parliament may extend the normal tenure of a state legislative assembly (five years) by one year each time
(for any length of time) during a national emergency, subject to a maximum period of six months after the
Emergency has ceased to operate.

Q 83.C
 Waste Minimisation Circle (WMC) programme of National Productivity Council (NPC) provides
information on waste and emission minimisation and recycling of waste.
 Statement 1 is correct: Waste Minimization is an appropriate strategy to address the problems of
industrial pollution. The scheme aims to assist the Small and Medium Scale Industries in adoption of
cleaner production practices.The objectives of the clean technology scheme are as follows:
o To develop and promote programmes for clean technologies

29 www.visionias.in ©Vision IAS


Join Our Telegram Channel https://t.me/UPSCMaterials For Instant Updates

o To develop tools and techniques for pollution prevention


o To formulate strategies and programmes in sustainable development.
o To prevent pollution from small scale industries. Activities relating to demonstration of already
proven cleaner technologies/techniques, preparation of sector-specific manuals on waste
minimisation, setting up of ‗Waste Minimisation Circles‘ in specific clusters of small scale industries,
training and awareness programmes for the personnel in small scale industries and waste minimisation
and demonstration studies in selected sector.
 Statements 2 and 3 are correct: Waste Minimisation Circle (WMC) is an initiative sponsored by the
Ministry of Environment and Forests, Government of India under the policy framework of promoting
waste minimisation in India. The formation of waste minimization circles was initiated in 1995-96
through the National Productivity Council (NPC) acting as the nodal agency and with support from the
World Bank, and is part of a project that aims to promote group efforts for demonstrating cleaner
production techniques, and to provide opportunities for sharing information and knowledge on pollution
prevention.

Q 84.B
 The annual State and Trends report, published by the World Bank, has established itself as an
important reference document—first on carbon markets and, later, on carbon pricing more broadly—by
providing readers with up-to-date information on developments in initiatives and policies around the
world.
 The 2018 edition of the report (State and Trends of Carbon Pricing 2018) focuses exclusively on data
and information on the evolving initiatives that put a price on carbon, in terms of their most current status
and emerging trends. It includes an expanded discussion on what the trends are telling us about the
underlying motivations of and the direction the world is moving in when it comes to carbon pricing.
 For the purpose of the report, carbon pricing refers to initiatives that put an explicit price on greenhouse
gas (GHG) emissions. This includes emissions trading systems, offset mechanisms, carbon taxes, and
results-based climate finance. Such initiatives are being planned and implemented at international,
regional, national, and subnational levels.
 Other policies that implicitly price GHG emissions, such as the removal of fossil fuel subsidies (which are
also sometimes referred to as ―negative carbon pricing‖), fuel taxation, support for renewable energy, and
energy efficiency certificate trading, are also necessary, but the report focuses on initiatives that put an
explicit price on emissions.

Q 85.B
 Dochala, also called ek-bangla is a style of temple architecture that arose in Bengal. The temples have a
curved roof with two sloping sides.
 Chauchala ("four caves") is a style of temple architecture that arose in Bengal. The temples have a curved
roof composed of four triangular segments.

Q 86.D
 Statement 1 is correct: A Non Performing asset (NPA) is a loan or advance for which the principal or
interest payment remained overdue for a period of 90 days. Banks are further required to classify NPAs
further into Substandard, Doubtful and Loss assets.
o Substandard assets: Assets which has remained NPA for a period less than or equal to 12 months.
o Doubtful assets: An asset would be classified as doubtful if it has remained in the substandard
category for a period of 12 months.
o Loss assets: As per RBI, ―Loss asset is considered uncollectible and of such little value that its
continuance as a bankable asset is not warranted, although there may be some salvage or recovery
value.‖
 Statement 2 is correct: It is mandatory before a loan account turns into a NPA that banks are required to
identify incipient stress in the account by creating three sub-categories under the Special Mention Account
category as given in the below:
o SMA-0 - Principal or interest payment not overdue for more than 30 days but account showing signs
of incipient stress (Please see Annex)

30 www.visionias.in ©Vision IAS


Join Our Telegram Channel https://t.me/UPSCMaterials For Instant Updates

o SMA-1 - Principal or interest payment overdue between 31-60 days


o SMA-2 - Principal or interest payment overdue between 61-90 days
 Statement 3 is correct: The parameters for identifying both NPA and Special Mention Accounts are
decided by Reserve Bank of India.

Q 87.D
 National Policy on Biofuels - 2018 was recently approved by Union Cabinet.
 Some of the salient features are as:
o The goal of the policy is to enable the availability of biofuels in the market thereby increasing its
blending percentage. Currently, the ethanol blending percentage in petrol is around 2.0% and
biodiesel blending percentage in diesel is less than 0.1%. An indicative target of 20% blending of
ethanol in petrol and 5% blending of biodiesel in diesel is proposed by 2030.
o The policy will also allow production of ethanol from damaged food grains like wheat, broken rice
etc. which are unfit for human consumption. During an agriculture crop year when there is projected
oversupply of food grains as anticipated by the Ministry of Agriculture & Farmers Welfare, the
policy will allow conversion of these surplus quantities of food grains to ethanol, based on the
approval of National Biofuel Coordination Committee proposed under this Policy.
o 100% Foreign Direct Investment (FDI) in biofuel technologies would be encouraged through
automatic approval route provided biofuel so produced is for domestic use only.
o As the domestic biofuels availability is much lower than the Country‘s requirement, the export of
biofuels will not be allowed.

Q 88.B
 A ―flyway‖ is the total area used by (groups of) populations or species of birds, throughout their annual
cycle, including the breeding areas, migration stop-over and non-breeding (wintering) sites.
 Many of these sites tend to be highly productive and are thus also of importance to non-migratory birds
and other biodiversity. In the staging and non-breeding areas of the flyway, the high productivity also
enables local people to benefit from food, shelter and water.
 The Central Asian Flyway (CAF) covers a large continental area of Eurasia between the Arctic and Indian
Oceans and the associated island chains. The Flyway comprises several important migration routes of
waterbird, most of which extend from the northernmost breeding grounds in Russia (Siberia) to the
southernmost non-breeding (wintering) grounds in West and South Asia, the Maldives and British Indian
Ocean Territory. The birds on their annual migration across the borders of several countries.
Geographically the flyway region covers 30 countries of North, Central and South Asia and Trans-
Caucasus.
 The Convention on Migratory Species convened three meetings of the central Asian flyway range
states (Tashkent 2001, New Delhi, 2005 and Abu Dhabi, 2012). The second meeting in New Delhi
(2005) broadly agreed on the content of the Central Asian Flyway Action Plan to Conserve
Migratory Waterbirds and their Habitats. The Action Plan was developed with technical support
from Wetlands International and was finally adopted in January 2008.
 The Central Asian Flyway Action Plan sets the agenda for enhanced regional environmental
cooperation among the Central Asian Flyway states to promote the conservation of migratory
waterbirds and their habitats. The Action Plan builds on and complements actions that are being
undertaken by national governments to promote conservation.
 India has a strategic role in the flyway, as it provides critical stopover sites to over 90% of the bird
species known to use this migratory route.
o The Indian National Action Plan (NAP) for the conservation of migratory birds and their habitats
states the national priority and specific actions required to ensure healthy populations of these
migratory species in India, within their range across the flyway. The NAP draws from the Central
Asian Flyway Action Plan.

Q 89.C
 Pokkali Paddy is a saltwater-tolerant paddy in the coastal fields of Alappuzha, Ernakulam and
Thrissur districts of Kerala. Various agriculturists raised their concern about the growing threat to local-

31 www.visionias.in ©Vision IAS


Join Our Telegram Channel https://t.me/UPSCMaterials For Instant Updates

community driven Pokkali Paddy cultivation. Unscientific and unseasonal prawn culture, high operation
cost etc. are some of the threats being faced by this variety.
 Pokkali cultivation is a traditional indigenous method of rice-fish rotational cultivation practiced in the
coastal belts. This GI-Tagged paddy is a single-season (only one yield in a year) paddy raised in saltwater
fields between June and November followed by a season of fish-farming. After the harvest, the paddy
stubble in the fields acts as food and shelter for shrimp and small fish. It is an example of how new stress
tolerant genotypes have evolved under natural selection pressure. To elaborate, in order to survive the
tides, the plant adapts itself by growing taller, around 1.5 meters.

Q 90.B
 The ‗Harmonised Master List of Infrastructure and its Sub Sectors‘ contains the list of sectors which have
been provided with the Infrastructure status. The list has 5 main infrastructure sectors and infrastructure
sub-sectors. This is a guide to all agencies involved in the field of infrastructural development or
financing in India.
 Statement 3 is not correct: The process for granting infrastructure status falls within the purview of the
Ministry of Finance. The institutional mechanism to update the Master List and for revisiting the sub-
sectors outside the Master List requires a review by a Committee chaired by Secretary, Department of
Economic Affairs. The recommendations of the Committee are then considered by the Union Finance
Minister.
 Statement 2 is correct: Some of the major sectors of the list include Airport, Railways, Telecom
Services, Logistics, Affordable Housing, etc.
 Statement 1 is correct: Granting a sector infrastructure status enables it to avail infrastructure lending at
easier terms with enhanced limits, access to larger amounts of funds as External Commercial Borrowings
(ECB), access to longer tenor funds from insurance companies and pension funds and be eligible to
borrow from India Infrastructure Financing Company Limited (IIFCL).

Q 91.C
 Chhau Dance: Traditionally performed by men, the main objective of this dance form is to
celebrate martial arts. However, there is religious significance as well and more often than not these
dances convey stories from Ramayana and Mahabharata. Chhau dance can be classified into three
divisions based on the place it is performed; Mayurbhanj Chau of Odisha, Seraikella Chau of Jharkhand
and Purulia Chau of Bengal. While the performers wear amazing masks in Jharkhand and Bengal, the
dancers in Odisha don‘t necessarily wear one while performing. Music flows in the form of percussion
instruments like ‗Dhol‘, ‗Nagara‘ and wind instruments like ‗Shehnai‘. Performing Chhau requires
tremendous stamina and strength as the energy levels maintained throughout the performance is pretty
high. The Chhau dance along with Mudiyettu, a ritual theatre of Kerala; the Kalbelia folk songs and
dances of Rajasthan was included in the UNESCO's Representative List of the Intangible Cultural
Heritage of Humanity in 2010.
 Bihu Dance: Bihu is a fast-paced, extremely joyful dance, hailing from the state of Assam. It is
performed by young girls and boys during the festivals of Bihu, Assam‘s three important agricultural
festivals. The dance is performed to a twin-faced drum, with one end played with a stick and the other
with the palm. A Bihu performance is usually pretty long and is full of exciting changes in rhythm, mood,
movements, pace, tempo and improvisation.
 Lavani: Traditionally performed to the beat of the dholki, Lavani is a high-energy performance (usually
by women), and has contributed immensely to the development of folk theatre in Maharashtra. There are
two types of Lavani performances - Phadachi Lavani (enacted in a public space, a theatrical atmosphere)
and Baithakachi Lavani (performed in a closed space to a select audience, and mostly while sitting down).
Lavani dancers wear bright and decadent looking Navvari sarees, tie their hair in a tight bun and are
adorned in stunning jewellery. A Lavani performance usually chronicles the story and elements of a man-
woman relationship.
 Ghoomar: Ghoomar is a traditional folk dance synonymous with the Indian state of Rajasthan. Ghoomar
is performed by women in colourful swirling ghagharas. The beauty of this dance is in the stunning
pirouetting which goes on to reveal the various gorgeous colours of the swirling skirts. The women also
clap and snap their fingers while dancing, at particular parts during the song. The dance is performed in
honour of the Goddess Saraswati.

32 www.visionias.in ©Vision IAS


Join Our Telegram Channel https://t.me/UPSCMaterials For Instant Updates

Q 92.C
 Statement 1 is not correct: Agenda 21 is a global action plan for sustainable development into the 21st
century. Agenda 21, the Rio Declaration on Environment and Development, and the Statement of
principles for the Sustainable Management of Forests were adopted by more than 178 Governments at
the United Nations Conference on Environment and Development (UNCED) held in Rio de Janeiro,
Brazil, 3 to 14 June 1992.
 Statement 2 is not correct: The Agenda is a non-binding programme of action, which was adopted by
more than 178 Governments at the 'Earth Summit' in 1992. Although the Agenda lacks the force of
international law, the adoption of the texts carries with it a strong moral obligation to ensure
implementation of the strategies.
 Statement 3 is correct: The implementation of the Agenda is primarily the responsibility of
governments, through national strategies, plans, policies and procedures. International and regional
organizations are also called upon to contribute to this effort. The broadest public participation and the
active involvement of non-governmental organizations and other groups are encouraged.

Q 93.B
 Statement 1 is correct: The National Social Conference was formed at the third session of the Indian
National Congress in 1887 to provide a forum for the discussion of social issues. The Bharat Mahila
Parishad was the women‟s wing of this and was inaugurated in 1905. It focused on child marriage,
condition of widows, dowry and other evil customs. In 1910, Sarala Devi Chaudhurani, daughter of
Swarnakumari Devi formed the Bharat Stree Mandal (Great Circle of India Women) with the object of
bringing together women of all castes, creeds, classes and parties on the basis of their common interest in
the moral and material progress of the women of India.
 Statement 2 is not correct: Women‟s Indian Association (WIA) was founded in 1917 by Annie
Besant, Margaret Cousins and Dorothy Jinarajadasa, all three Irish women Theosophists, who had
been suffragettes in their own country. They were joined by Malati Patwardhan, Ammu Swaminathan,
Mrs. Dadabhoy and Mrs. Ambujammal. WIA was in a sense the first all India women‘s association with
the clear objective of securing voting rights for women.
 Statement 3 is not correct: The All India Women‟s Conference (AIWC) was established in 1927 at
the initiative of Margaret Cousins to take up the problem of women‟s education. Women from
different parts of India belonging to different religions, castes and communities attended the first session
in Pune that was a great success. AIWC‘s initial concern was with education but it realized that girls did
not go to school because of purdah, child marriage, and other social customs. It therefore took up these
issues. It waged a vigorous campaign for raising the age of marriage which led to the passing of the Sarda
Act in 1929.

Q 94.B
 Statement 1 is not correct: There shall be a Council of Ministers with the Prime Minister at the head to
aid and advise the President who shall, in the exercise of his functions, act in accordance with such
advice. The total number of ministers, including the Prime Minister, in the Council of Ministers shall not
exceed 15% of the total strength of the Lok Sabha. This provision was added by the 91st Amendment Act
of 2003.
 Statement 2 is correct: Article 74(2) of the Indian Constitution says "The question whether any and if so
what, advice was tendered by Ministers to the President shall not be inquired into in any court".
 Statement 3 is correct: In India, there is no provision in the Constitution for the system of legal
responsibility of a minister. It is not required that an order of the President for a public act should be
countersigned by a minister.
Q 95.A
 Statements 2 and 3 are not correct: Financial Sector Regulator Appointment Search Committee
(FSRAC) was set up based on the recommendation of the Financial Sector Legislative Reforms
Commission (headed by Justice B. N. Srikrishna). It is headed by the Cabinet Secretary P K Sinha. Apart
from him, the panel includes Additional Principal Secretary to the Prime Minister P K Mishra, who is a
permanent government nominee, and three other experts, among others. P. J. Nayak Committee was
formed to review Governance of Boards of Banks in India
 Statement 1 is correct: It is a standing committee which is tasked to recommend suitable persons for
selection of Chairperson and whole time and part-time members of the financial sector regulators.

33 www.visionias.in ©Vision IAS


Join Our Telegram Channel https://t.me/UPSCMaterials For Instant Updates

Q 96.A
 Statement 1 is correct: The RIN revolt started on 18 February when 1100 naval ratings of HMIS Talwar
struck work at Bombay to protest against the treatment meted out to them — flagrant racial
discrimination, unpalatable food and abuses to boot.The arrest of B.C. Dutt, a rating, for scrawling
‗Quit India‘ on the HMIS Talwar, was sorely resented.
 Statement 2 is not correct: From the initial flashpoint in Bombay, the revolt spread and found
support throughout British India, from Karachi to Calcutta, and ultimately came to involve over
20,000 sailors in 78 ships and shore establishments. Karachi was a major centre, second only to Bombay.
The news reached Karachi on 19 February, upon which the HMIS Hindustan along with one more ship
and three shore establishments, went on a lightning strike. Sympathetic token strikes took place in military
establishments in Madras. Vishakhapatnam. Calcutta, Delhi, Cochin, Jamnagar, the Andamans, Bahrain
and Aden Seventy eight ships and 20 shore establishments, involving 20,000 ratings, were affected.
 Statement 3 is not correct: The mutineers in the armed forces got no support from the national leaders
and were largely leaderless. Mahatma Gandhi condemned the riots and the ratings' revolt. His
statement on 3 March 1946 criticized the strikers for revolting without the call of a "prepared
revolutionary party" and without the "guidance and intervention" of "political leaders of their choice".
Q 97.D
 'Insect Allies‘, launched by the Defence Advanced Research Projects Agency (DARPA), which is
responsible for developing military technologies in the U.S. The project involves using gene-editing
techniques like CRISPR to infect insects with modified viruses that could help make America's crops
more resilient. This is being done ostensibly to address infections.

Q 98.C
 Sikkim is an Indian state, situated in the Eastern Himalayas spread below the world's third highest
mountain Kangchendzonga (8585m) revered by the Sikkimese as their protective deity. It is located
between the latitude of 27˚ 03' and 28˚ 07' North latitude. Haryana is located between 27°39' to
30°35' N latitude. Hence both states pass through the same latitudes.
 Sikkim is separated by the Singalila range from Nepal in the west, Chola range from Tibet in the
northeast and Bhutan in the southeast.
 Rangit and Rangpo rivers form the borders with the Indian state of West Bengal in the south.
 Sikkim is the first organic state in the world. All of its farmland is certified organic. At the same time,
Sikkim‘s approach reaches beyond organic production and has proven truly transformational for the state
and its citizens.

Q 99.B
 Statements 1 and 2 are correct: The Agreement on Government Procurement (GPA) is a plurilateral
agreement under the auspices of the World Trade Organization (WTO) that entered into force in 1981. It
was then renegotiated in parallel with the Uruguay Round in 1994, and entered into force on 1 January
1996. It regulates the government procurement of goods and services by the public authorities of the
parties to the agreement, based on the principles of openness, transparency and non-discrimination.
 Statement 3 is not correct: The major parties to the agreement include European Union, Japan, United
States, Israel etc. India has observer status and is not a party to the agreement.

Q 100.A
 Statement 1 is correct: An ISBN is an International Standard Book Number. It is a product that is used
by publishers, booksellers and libraries for ordering, listing and stock control purposes. It enables
them to identify a particular publisher and allows the publisher to identify a specific edition of a specific
title in a specific format within their output.
 Statement 2 is not correct: There is no legal requirement for an ISBN and it conveys no legal or
copyright protection. It is a product number. ISBNs are assigned to publishers in the country where the
publisher's main office is based/publishing activity takes place. This is irrespective of the language of the
publication or the intended market for the book.

Copyright © by Vision IAS


All rights are reserved. No part of this document may be reproduced, stored in a retrieval system or transmitted
in any form or by any means, electronic, mechanical, photocopying, recording or otherwise, without prior
permission of Vision IAS

34 www.visionias.in ©Vision IAS

You might also like